Series 7 - Review Q's

Réussis tes devoirs et examens dès maintenant avec Quizwiz!

If a husband makes a gift of $100,000 to his wife, a U.S. citizen, how much of the gift is subject to gift taxes?

$0 Interspousal gifts to citizens of the United States, regardless of amount, are not subject to gift taxes.

On December 13, an investor buys six ABC Feb 60 calls at 2.25 each, when ABC is trading at 59.50 per share. If the calls expire unexercised, how much money will the investor lose?

$1,350 Buyers of options lose premiums if the options expire unexercised. The most this investor can lose is the number of contracts (6) multiplied by the amount of the premium (2.25). This investor's maximum loss is $1,350.

A customer purchases an ABC 6½% convertible preferred stock at $80. The conversion price is $20. If the common stock is trading 2 points below parity, the price of ABC common is

$14 The conversion ratio is computed by dividing par value by the conversion price ($100 par ÷ $20 = 5). Parity price of the common stock is computed by dividing the market price of the convertible by the conversion ratio ($80 ÷ 5 = $16). $16 − 2 = $14.

An affiliate holding restricted stock wishes to sell shares under Rule 144. He has held the shares, fully paid, for six months, and the issuer has 2.4 million outstanding shares. Form 144 is filed on Monday, April 10, and the average weekly trading volume for the past four weeks is 24,500 shares per week. The maximum number of shares the customer can sell with this filing is

24,500. Under Rule 144, after holding the fully paid restricted shares for six months, the affiliate can begin selling. For affiliates, volume restrictions always apply. They can sell the greater of 1% of the total shares outstanding or the weekly average of the past four weeks' trading volume (the four weeks preceding the Form 144 filing). In this case, 1% of the total shares outstanding is 24,000 (1% × 2.4 million). The weekly average of the past four weeks' trading volume is 24,500. Therefore, the most the affiliate can sell during the 90 days following the Form 144 filing is 24,500 shares.

A customer who sold a bond at a loss must wait how long before she can buy back a substantially identical bond and not have the sale classified as a wash sale?

30 days According to the wash sale rule, a customer who sells a security at a loss cannot buy back a substantially identical security 30 days before or 30 days after the sale that established the loss.

A registered representative has just been disciplined under the Code of Procedure. Within how many days does the U-4 need to be updated?

30 days The U-4 must be updated within 30 day of a qualifying event, such as a change to the residence address or a disciplinary action.

A customer contacted her registered representative requesting that her account be updated with her new residential address. The member firm must send a copy of the updated account record to the customer within

30 days for any changes made

An investor is long 500 shares of DEFG common stock, short 200 shares of DEFG common stock, and short 300 shares of DEFG 5% preferred stock. A tender offer for DEFG common shares is announced. Under SEC rules, this investor is permitted to tender

300 shares. The rules permit tendering of net long shares. That means the difference between the long and short positions. In this question, the investor is net long 300 shares: the difference between the 500 long and the 200 short. The preferred shares are not relevant because the tender is only for the common stock.

An abstract of a municipal securities issue official statement must be maintained on file for how long?

4 years The Municipal Securities Rulemaking Board requires firms to retain abstracts of official statements for four years—the same as all pieces intended to communicate with the public.

Based on yesterday's closing price of $60 per share, Blech Sheet Metal, Inc., has a current P/E ratio of 12:1. If the current quarterly dividend payment is $0.50 per share, the dividend payout ratio is

40% With a price-to-earnings ratio of 12:1, the earnings per share (EPS) is $5.00 ($60 divided by 12). Four quarterly dividends of $0.50 is an annual dividend of $2.00 per share. If the company is paying $2 per share from the $5 per share earnings, that is a dividend payout ratio of 40%.

Your client sells one naked MAV Oct 40 call at 2 when the market price of MAV is $41. What must MAV be selling at for the client to break even?

42 The breakeven point for a call is the strike price plus the premium (call up). The breakeven point is the same for both the buyer and the writer.

A major risk associated with investing in DPPs is the lack of liquidity. What steps could the program sponsor take that could have the effect of increasing the liquidity of an existing program?

A DPP rollup is a transaction involving the combination or reorganization of one or more limited partnerships into securities of a successor corporation. The securities of the successor corporation would likely have greater liquidity. This would have the effect of turning the illiquid DPP into more liquid securities. Disclosure documents must be provided to investors prior to the transaction disclosing risk, the GPs opinion regarding fairness of transaction, and reports and appraisals in connection with the transaction.

An investor buys a yield-based Sep 70 call on a 30-year T-bond for a premium of 2.50. At expiration, if the yield on the most recently issued T-bond is 7.95%, what is the investor's gain or loss?

A Sep 70 call means that the holder is buying a 7% yield. The investor can close the option at its intrinsic value (7.95 − 7.00 = 0.95; 0.95 × 10 × $100 = $950 received upon close). Subtract the $250 premium paid for a total profit of $700.

Under the conduit theory of taxation, which of the following statements are true?

A fund is not taxed on earnings it distributes if it distributes at least 90% of its net investment income. Investors are taxed on earnings they receive in cash.

A fund seeks maximum capital appreciation by investing in common stocks of companies located outside the United States. The management selects well-established companies that are listed on their national stock exchanges and that have demonstrated high earnings potential. This information describes which of the following mutual funds?

ATF Overseas Opportunities Fund descriptions at the end of the notebook

Which of the following individuals could most likely open an account at a FINRA member firm without notifying or receiving permission from their employer?

An individual who sells only fixed annuities Whenever an employee of a FINRA member wants to open a securities account with another FINRA member firm or financial institution, the employee must give prior written notice to his employer and receive prior written consent from his employer before the account can be opened. Someone selling fixed annuities only is most likely not associated with a member firm, while anyone selling variable annuities must be.

Which of the following would not be considered institutional communications with the public?

An internal memo promoting a new product that will be offered to your firm's institutional customers only Institutional communications specifically exclude internal communications. Communications with another member firm, a government entity such as a municipality, or with someone designated to act on behalf of one of your firm's institutional customers would all fall within the definition of institutional communications.

All of the following securities are issued at a discount except

CDs CDs are interest-bearing debt instruments issued by banks at their face value. All of the others are issued at a discount.

A type of alternative trading system that trades listed stocks and is required to register with the SEC as a broker-dealer is

Electronic communication networks (ECNs) are a type of alternative trading system (ATS) that trade listed stocks and other exchange-traded products

What might happen if a limited partner begins making business decisions for the partnership?

He might jeopardize his limited liability status. If a limited partner has control over the partnership operation (i.e., he makes partnership decisions), he could be judged a general partner and, thus, have unlimited liability.

A registered representative is opening both cash and margin accounts for a corporation. Which of the following documents will he need? The corporation's charter and bylaws A copy of the corporation's most recent balance sheet The corporation's last three profit and loss statements The name(s) of natural persons authorized to trade the account

I and IV Corporate accounts are generally those established by the officers of a corporation. Such accounts require a copy of the corporate resolution naming the authorized person(s) and account trading limits (if any). If it is to be a margin account, a copy of the corporate charter and a signed margin agreement are also required.

A customer buys 100 shares of ABC at 56.50 and writes 1 ABC Aug 60 call at 2. If the call is exercised, the consequences are a cost basis of $56.50 per share. a cost basis of $58.50 per share. sales proceeds of $60 per share. sales proceeds of $62 per share.

I and IV The premium of the option affects the basis of the stock (bought or sold) as a result of exercise, adding the premium per share ($2) to the price per share ($60), for total sales proceeds of $62. The original cost basis is not affected by the exercise, so it remains $56.50.

A customer writes 1 ABC Jan 65 put at 6 and 1 ABC Jan 55 call at 7 when ABC is trading at 60. This position will be profitable if, at expiration, ABC is above $68. at $60. below $52. between $52 and $68.

II and IV This is a short combination where both contracts are in the money. Short combinations are like short straddles, except the strike prices and/or the expirations are different. The breakeven points are computed by adding the combined premiums to the strike price of the call (strike price + premium) and subtracting the combined premiums from the strike price of the put (strike price − premium). The breakeven points, therefore, are 52 (65 − 13) and 68 (55 + 13). With a short combination, the investor makes money if the stock stays inside the breakeven points.

When a registered representative opens an options account for a new client, in which order must the following actions take place? Obtain approval from the branch manager Obtain essential facts from the customer Obtain a signed options agreement Enter the initial order

II, I, IV, III The steps in opening a new options account occur in the following order: obtain essential facts about the customer, have the manager approve the account, enter the initial order, and have the customer sign the options agreement within 15 days.

Which of the following investments is most suitable for an investor seeking monthly income?

Money market mutual fund

Which of the following municipal bonds may be paid by a state's legislative apportionment of funds to service the debt?

Moral obligation If a moral obligation bond goes into default, bondholders do not have the right to sue to force a tax to pay off the bonds. The only way bondholders can recover the principal is through legislative apportionment. The issuer's legislative body has to appropriate funds to pay off the bonds. With a moral obligation bond, issuers have the moral, but not legal, obligation to service the debt.

Regarding the taxation of dividends received from corporate securities, which of the following are true?

Nonqualified dividends are taxed at the rate the investor's ordinary income will be taxed. Nonqualified dividends are not taxed. Qualified dividends are taxed at a maximum rate specified by the IRS and will depend on the investor's income tax bracket. Qualified dividends are taxed at the rate the investor's ordinary income will be taxed.

Which of the following are directly backed by the U.S. government?

PHAs and NHAs Public Housing Authority and New Housing Authority issues are unique as municipal instruments because they are fully backed by the U.S. government.

For tax-reporting purposes, qualified dividends are considered to be what type of income?

Portfolio income includes dividends (qualified or not), interest, and net capital gains derived from the sale of securities.

From first to last, which of the following sequences reflects the priority of payments made when a limited partnership is liquidated?

Secured creditors, general creditors, limited partners, and general partners. Creditors are paid first in a liquidation, with priority given to the secured lenders; general partners are the last to be paid.

Which of the following would establish a covered put?

Short stock at 40, short put at 35 A covered put is created when a short stock is combined with a short put. Covered puts are established when the investor is neutral or slightly bearish; therefore, the strike price of the put is less than the cost of the stock sold short. The reason the put writer is covered (protected) is that if the stock's price should decline below 35 and the holder of the option exercises, the stock purchased by the writer is used to cover (replace) the borrowed stock for the short sale at 40.

Which of the following is an example of sovereign debt?

Sovereign debt represents loans to governments. On the exam, it is likely that the examples will be foreign governments, not U.S. Treasury securities. The Royal Bank of Canada is a privately owned corporation and its debts are not those of the Canadian government. Bank of England notes are the paper currency issued (e.g., the ₤10 and ₤20 notes).

The Bond Buyer compiles several indexes of municipal bonds. Which of the following is limited to bonds with the highest ratings?

The 11 Bond Index This index takes the highest rated (AA or better) bonds from the 20 Bond Index of bonds with A ratings or better.

If a customer writes 2 ABC Feb 90 puts at 8 and buys 2 ABC Feb 80 puts at 2, which of the following statements are true?

The spread is bullish. The breakeven point is 84. This is a credit put spread (the net credit being 6 points per share) in which the breakeven point is calculated by subtracting the net premium (debit or credit) from the higher strike price (90 - 6 = 84). A credit put spread is like the net sale of a put, and buying the lower strike price in any spread (put or call) is bullish.

Certain investments are available only to those who meet the SEC's definition of an accredited investor. Which of the following qualify?

Those using the net worth standard to qualify as an accredited investor must exclude the equity in their primary residence. Those using the income standard must reach levels in excess of $200,000 as individuals and in excess of $300,000 when combining with the income of a spouse. Those earnings must have been achieved in the previous two years along with an expectation of similar earnings for the current year.

A client writes 1 Jan 60 put and buys 1 Jan 50 put. This is

a credit bull spread; the investor wants the price to stay above 60. This is a put credit spread, and bulls sell puts. The 60 put is worth more because it has a higher strike price. Long the lower put is bullish; short the lower put is bearish.

All of the following will cause a change in SMA in a long account except

a decrease in market value. Once SMA is created in a long account, it is not reduced by a decline in market value. The SMA may still be taken out, provided it will not pull the account below the maintenance level. An increase in market value or a sale of stock increases SMA. The purchase of stock decreases available SMA.

In a scheduled premium variable life insurance policy, all of the following are guaranteed except

a minimum cash value. In a variable life insurance policy, a minimum death benefit is guaranteed, but no cash value is guaranteed. There is a contract exchange privilege during the first 24 months, allowing the conversion of the variable policy to a comparable form of permanent insurance, and the 75% cash value loan minimum applies after the third year of coverage.

An investor receiving a quote of 102 for a municipal security is probably interested in

a term bond. A quote of 102 is referred to as a dollar quote ($1,020) rather than a yield quote. The most common dollar bonds are those with a term maturity. The other choices are most often quoted on a yield basis rather than a price basis.

If ABC Corporation reports a loss for the year, it is obligated to pay interest on all of the following except

adjustment bonds. Even if a corporation reports a loss, the corporation is obligated to pay interest on all of its outstanding debt except for income (adjustment) bonds. Adjustment bonds require interest to be paid only if ABC has sufficient earnings and the payment is declared by the board of directors.

A customer has realized a capital gain from the sale of a municipal bond. To reduce her tax liability, the capital gain can be offset against a capital loss in which of the following investments? General obligations Equity securities Corporate bonds Collateralized mortgage obligation

all Any capital loss will offset a capital gain.

If a customer buys a municipal bond at 110, maturing in eight years, but sells the bond six years later at 103½, the customer will have

a $10 per bond gain. Municipal bonds that are purchased at a premium must be amortized. This bond has a premium of $100, which over eight years, amounts to $12.50 per year. The cost basis of the bond at the time of the sale is $1,100 − (6 × $12.50), or $1,025. If the bond is sold for $1,035, the customer has a gain of $10 per bond.

Under SEC Rule 10b-10, customer confirmations must be sent at or before

completion of a transaction. Rule 10b-10 is the SEC's rule on delivery of customer confirmations, and it requires that they be sent no later than the completion of the trade (synonymous with the settlement date). The settlement date plus two business days refers to Regulation T payment.

An example of overlapping debt would be a school district and

county general debt. Do not combine revenue bonds with GOs to determine overlapping debt. Overlapping debt occurs in real estate taxing situations. Only GOs are backed by real estate taxes.

If a customer buys 100 shares of stock and writes one out-of-the-money call against her long position, the breakeven point is

the cost of stock purchased less premium. When the investor owns stock and sells a call, the call is covered. Breakeven is computed by subtracting the premium from the stock's purchase price.

All of the following would be found in a bond resolution for a new municipal issue except

the costs to be incurred by the issuer in connection with the offering. The bond resolution (or the bond contract) spells out the characteristics of the issue (maturities, call features, etc.), the issuer's responsibilities to bondholders, and any restrictive covenants to which the issuer must adhere. Costs to be incurred by the issuer have no impact on bondholders.

An important feature of scheduled premium variable life insurance policies is that

the death benefit can never fall below the guaranteed minimum amount. Scheduled (fixed) premium variable life always has a guaranteed death benefit. Cash values cannot be guaranteed, only the death benefit. Better than expected performance of the separate account will lead to increased cash values, but it will not affect the premiums.

Maintaining a fair and orderly market on the NYSE trading floor is the responsibility of

the designated market maker. The designated market maker maintains a fair and orderly market on the NYSE exchange floor. The role historically was filled by the specialist, a term that might still appear on the exam. Floor brokers represent member firms in the execution of their customer orders.

In portfolio theory, the alpha of a security or a portfolio is

the difference between the expected return of the portfolio, given the portfolio's beta, and the actual return the portfolio achieved. The higher the alpha, the better the portfolio has done in achieving excess or abnormal returns. The risk of the portfolio associated with the factors that affect all risky assets is systematic risk.

Limited partnership programs are categorized as direct participation programs. The term direct participation refers to

the flow-through of profits and losses of the partnership to the individual limited partners. Understanding the flow-through concept is critical with DPPs. Only DPPs allow flow-through of losses.

A customer asks for sales literature for a money market mutual fund. Upon receiving the literature, she notices

the fund seeks to maintain a stable price but the fund can lose money. Sales literature for money market mutual funds must include the fact that it is possible to lose money when investing in the fund.

If a father makes a gift of securities to his 10-year-old daughter, gift taxes would be based on

the market value of the securities on the date of gift. If a gift tax is due, it is paid by the donor and based on the gift's value on the date it is given.

With regard to a variable annuity, all of the following may vary except

the number of annuity units. During the accumulation phase, the number of accumulation units will increase as additional money is invested. When the contract is annuitized, the annuitant is credited with a fixed number of annuity units. Once annuitized, the number of annuity units does not vary. The value of accumulation and annuity units varies with the investment performance of the separate account.

Perhaps the most important thing to understand when a business is organized as a sole proprietorship is that

the owner is liable for all the debts of the business.

KAPCO Manufacturing Corporation declares a 5-for-1 stock split on its outstanding shares of $20 par value common stock. This split will cause

the par value of the shares to change to $4 per share. Whenever there is a stock split (forward, such as this, or reverse), the par value is adjusted for the split. With a $20 par and a 5-for-1 split, the par value now becomes $4 per share.

The rights and liabilities of general and limited partners are listed in

the partnership agreement. The agreement is the contract between the general and limited partners, and it contains each entity's rights and duties. The certificate of partnership is the document legally establishing the enterprise.

Limited partners in a real estate partnership have all of the following rights except

the right to decide which properties the partnership purchases. The limited partners have the right to inspect partnership records and the right to sue a general partner who acts outside the partnership agreement. The general partner normally sets her own compensation in the original agreement and makes all management decisions relative to the partnership's interests

A customer opens a margin account with a broker-dealer and signs a loan consent agreement. The loan consent agreement allows the firm to

loan out the customer's margin securities.

A structured instrument known as an asset-backed security would not be backed by

loans on marginable securities. One common theme uniting asset-backed securities is the contractual obligation to make payments. In the case of a margin account, there is no repayment schedule. The margin debt can exist for years with the only payment being that of interest.

An investor might expect to receive the greatest gain on an investment in a corporate bond by purchasing

long-term bonds when interest rates are high. If an investor purchases bonds when market interest rates are high, a drop in interest rates will lead to a corresponding increase in bond value. Long-term debt instruments will fluctuate to a greater degree than those with short-term interest rates. Thus, long-term debt offers the greater chance of gain.

An investor establishes the following positions: Long 1 XYZ Apr 40 call for 6 Long 1 XYZ Apr 50 put for 8 If both options are sold for intrinsic value when XYZ trades at 44, the investor realizes a loss of

$400 If the opening purchase of the XYZ Apr 40 call was made at 6, and the closing sale of that call was made at 4, the difference of 2 represents a $200 loss. If the opening purchase of the XYZ Apr 50 put was made at 8, and the closing sale of that put was made at 6, the difference of 2 represents a $200 loss. The total loss for the account is $400.

If an investor buys one KLP Oct 95 put at 6.50, what is the investor's maximum potential gain?

$8,850 The maximum gain on a long put is calculated by subtracting the premium from the strike price (95 − 6.50 = 88.50 per share). One contract represents 100 shares, so the buyer's maximum gain is $8,850 if the stock declines to zero. Because put buyers are bearish, they will make money if the stock falls below the breakeven point of 88.50.

If a customer opens a spread on Canadian dollars (10,000 units) by purchasing 1 Dec 74 call for 2.30 and selling 1 Dec 77 call for 1.50, what is the total cost of this debit spread?

$80 The customer pays a premium of 2.30 ($230) and receives a premium of 1.50 ($150) for a net debit of 0.80, or $80.

ALFA Electronics has been trading around 70. A customer tells his registered representative that if 1,000 shares of the stock can be purchased in a single attempt, the customer will take it. If not, the customer is not interested and the order should be canceled immediately. How should the representative enter this order?

1,000 ALFA FOK at 70 A fill-or-kill (FOK) order designates that the customer wants the order to be filled in its entirety in one attempt or be canceled. With an all-or-none (AON) order, the broker-dealer can make numerous attempts to fill the order in its entirety. With an immediate-or-cancel (IOC) order, the broker-dealer can make only one attempt to fill the order, but a partial fill is acceptable.

If two customers are tenants in common in a joint account, which of the following statements are true?

1. They need not make equal investments in the account. 2. They need not have equal interests in the property in the account.

If a contract calls for a delivery between member broker-dealers of 500 shares of stock, all of the following certificate combinations would be a good delivery conforming with the FINRA Uniform Practice Code except

10 certificates for 30 shares each and 4 certificates for 50 shares each. Good delivery between member firms requires delivery of certificates of 100 shares, in multiples of 100, or the ability to put certificates of less than 100 shares into stacks of 100 shares. Note that the 10 certificates, each for 30 shares, could not be stacked in units of 100.

To meet a Fed call, a customer must deposit which of the following?

100% of the call in cash and 200% of the call in marginable securities To meet an initial call, the customer can deposit cash equal to the call or deposit fully paid-for marginable securities with a market value equal to twice the call. For example, if the trade is for $10,000, the margin call will be 50% of that, or $5,000. If the customer wishes to deposit fully paid-for marginable securities, it has to be in an amount sufficient for the BD to lend the account $5,000. With Regulation T at 50%, $10,000 of free and clear stock has a loan value of $5,000—exactly what is needed for the margin call.

A customer has placed an open order to buy 1,600 shares of GHI at $60. GHI declares a 25% stock dividend. On the ex-date, this order is considered a buy limit order for

2,000 shares at $48. The order is adjusted on the ex-date. The number of shares increases by the percentage of the stock dividend, and the specified price is reduced to compensate. In this case, the number of shares increased to 2,000 (1,600 + 25%), and the specified price is adjusted to $48 per share. To get the adjusted price, divide the total value of the original market order (1,600 × $60 = $96,000) by the new number of shares ($96,000 ÷ 2,000 = $48). The order's total market value, $96,000, remains the same. you can also mult. $60 x 1600 = 96k 2000 x$48 =96k they both have to equal the same

In general, commercial paper, a popular money market instrument, has a maturity not exceeding

270 days

Your customer owns a leveraged ETF having a performance goal of 200% of the underlying index. When purchased two days ago, the ETF was priced at 50. If the index was down 10% the first day and up 20% the second day, what is the value of the ETF today if it performed as it was intended to?

56 Priced at 50 when purchased, after the first day's 10% decrease in the index, the 2× leveraged ETF would be down 20% (20% × 50 = 10-point decrease) to 40. After the second day's 20% increase in the index, the 2× leveraged ETF would be up 40% (40% × $40 = 15-point increase) to 56. On the calculator provided at the testing center, you could arrive at the correct answer by taking the original 50, multiplying times 80% (the 10% decrease is doubled means the stock is only 80% of what it was) and then multiplying that result by 140% (2× the 20% increase).

A customer wrote 10 KLM Jun 80 calls for a premium of 4.75 at a time when the market value of KLM was 81.75. What is his gain or loss if he now closes out his positions at 2.12?

A $2,630 gain If the customer sold at 4.75 and purchased at 2.12, he nets 2.63, which is multiplied by 100 to yield a $263 gain per contract: 10 × $263 = $2,630 total gain.

Which of the following investments is most likely to have extension risk?

A CMO Extension risk is the uncertainty that a debt will be paid off ahead of schedule. This happens most frequently with mortgage-backed securities, such as CMOs. A CMO's yield and maturity are estimates based on historical data or projections of mortgage prepayments from the Public Securities Association (PSA). When that estimate is incorrect and the mortgage prepayments decline (such as in a period of rising interest rates), yields turn out to be lower than projected. Bonds do not have their maturity dates extended, so there is no extension risk.

Which of the following securities can be traded in the third market?

A closed-end investment company and an ETF The third market is defined as the trading of listed securities over the counter. There is no secondary market trading in open-end investment companies and unit investment trusts. Although most exchange-traded funds (ETFs) are structured as open-end companies, unless something in the question is dealing with the ETF's structure, open-end will always refer to a mutual fund.

Which of the following methods of real estate investing is a flow-through vehicle?

A real estate limited partnership (RELP)

A 45-year-old client of yours receives an inheritance of $100,000 and wishes to invest it without having to worry about any taxes being due until she reaches age 68. In addition, the client would like to have some protection against inflation. Which of the following would be the most appropriate recommendation?

A single premium deferred variable annuity There are two benefits to the deferred variable annuity. The first is that taxes on all earnings are deferred until withdrawal. The second is that, if the proper separate account subaccounts are selected, there is potential inflation protection.

An investor is concerned about safety. When consulting the ratings, which of the following securities would appear to be least likely to default on its obligation to make timely payment of interest and principal?

AA rated debenture

A sophisticated client has expressed an interest in becoming more aggressive with their investment strategy. Her current portfolio consists of the following: $50,000 cash $200,000 in retirement accounts $100,000 in various individual stocks in different industries $100,000 in a balance fund She is willing to invest $25,000 for a minimum of 7 to 10 years and accepts that the investment can and will fluctuate in value over time. Which of the following investments would be the most appropriate?

ABC Capital Appreciation Small-Cap Fund

Who signs the agreement among underwriters for a municipal bond issue?

All members of the underwriting syndicate including the managing underwriter, sign the agreement among underwriters. It is not signed by the issuer, bond counsel, or trustee.

Your customer, age 60, is retired and living at home with a fully paid-off mortgage. Her portfolio contains growth stocks and high-quality bonds, and she is a long-time investor and comfortable with moderate risk. Her objective is a moderate level of current income to supplement her corporate pension plan distributions and the earnings from her traditional IRA. How are the distributions taxed from her IRA?

All taxable distributions from a traditional IRA are taxed as ordinary income.

A customer purchases five 6.25% U.S. Treasury notes at 98.24. How much will the customer receive on each interest payment date?

Although minimum purchase denominations can be less, always use par value ($1,000) for these calculations. A 6.25% bond pays $62.50 annually (6.25% × $1,000 = $62.50). Therefore, a customer purchasing five bonds receives $312.50 each year. Because Treasury notes pay semiannually, each interest payment equals $156.25.

Which of the following is not a right conferred upon ownership of common stock?

Although ownership of common stock means the holder's maximum loss is limited to the original investment, it is not a stockholder right.

One of your customers called you with the good news that they are new grandparents. They are looking for a way to provide funds for the new child's college education and would like some kind of tax break if possible. What would be the most suitable suggestion?

Among the many benefits of the Section 529 plan is that all earnings between now and withdrawal can be tax free when used for qualified expenses. In addition, if the grandparents have a substantial estate, they can contribute up to $75,000 ($150,000 if a joint gift) without any gift tax ramifications.

Investing in undeveloped land satisfies which of the following primary objectives?

Appreciation

One of your customers has made periodic purchases of shares of the Castel Growth Fund over the past several years. The customer has decided to take a profit and sell some of those shares. When the investor's tax return is prepared for the year in which the sale of those shares occurs, it is necessary to establish a cost basis of the shares sold. Which of the following methods is available for mutual funds, that is not available for determining the cost basis of stock?

Average cost basis The Internal Revenue Service allows using the average cost basis to determine the cost basis of redeemed mutual fund shares. Investors cannot use this method when selling shares of any security other than a mutual fund. The other methods of determining cost basis are FIFO and share identification. FIFO is the default method used by the IRS if an investor fails to choose. Share identification can frequently result in a lower tax bill, especially if the security was purchased at different intervals at varying prices.

All of the following regarding Section 529 education savings plans are true

Contributions are made with after-tax dollars and are not deductible.

A registered representative is explaining the characteristics of a Coverdell Education Savings Account (ESA) to a customer. Which of the following statements regarding this type of savings account is correct?

Contributions to a Coverdell ESA are made with after-tax dollars so not tax deductible. Distributions used for qualified educational expenses are tax free.

Under the rules on communication with the public, review of which of the following by a principal may take place either before or after distribution?

Correspondence to 25 or fewer retail investors within any 30-calendar-day period A principal may review correspondence before or after its distribution as long as it is limited to no more than 25 retail investors during any 30-calendar-day period. The other choices require principal approval rather than just review before first use.

If your client wished to purchase a preferred stock that would offer him the highest likelihood of assured income, plus the opportunity to take part in the growth of the company's common stock, which of these features might he consider?

Cumulative and convertible The convertible feature allows the investor to take part in the company's growth because the price of the common stock will reflect that growth. The cumulative feature, although not guaranteeing dividends, means that any missed dividends will have to be paid before anything can be paid to the common shareholders.

Liquidity ratios measure the solvency of a firm, or the firm's ability to meet short-term financial obligations. Which of the following is a liquidity ratio?

Current assets divided by current liabilities

Which of the following is used to report a bankruptcy filing of a registered representative of a FINRA member firm?

DRP DRP stands for disclosure reporting page. That is the part of the Form U4 where a registered person indicates certain mandatory disclosures. In addition to bankruptcy filing, it includes the following: · Criminal disclosures · Civil disclosures related to nonsecurities cases · Disclosures of actions taken by regulatory authorities · Customer complaints · Termination for cause The TIF is the transfer initiation form used with ACATS to transfer a customer account from one member firm to another. ADV is the form used by investment advisers to register with the SEC or the appropriate state(s). There is no Form BF.

Debt service is best described as

Debt service is the total of interest and principal payable by the issuer plus any amount required to be deposited into a sinking or surplus fund.

Under ERISA, all of the following retirement plans must set standards for vesting, eligibility, and funding except

Deferred compensation plans are not qualified plans and may be discriminatory

Which of the following statements regarding Roth IRAs are true?

Earnings accumulate tax free. Distributions are not taxable if a holding period and age requirement are satisfied.

Which of the following securities would most likely have the lowest expense ratio?

Exchange-traded fund Generally, most exchange-traded funds (ETFs) have a lower expense ratio than comparable mutual funds. Variable annuity expense ratios tend to be higher than mutual funds, and the expense ratio for closed-end funds is similar to that of open-end (mutual) funds.

Which of the following statements regarding callable municipal bonds are true? Call premiums tend to increase over time. Call premiums tend to decrease over time. Call prices are stated as a percentage of the principal amount to be called. Call prices are stated as a percentage of the market value of the bonds to be called.

II and III Call premiums tend to decrease over time. The longer a customer has to hold the bond (and receive semiannual interest), the less of a premium an issuer will pay to take away the bond before maturity. Call prices are always stated as a percentage of the principal amount (par) to be called. For example, a call price of 103 means the issuer will pay $1,030 for each bond called.

Investors who are subject to AMT must have which of the following preference items added to adjusted gross income to calculate their tax liability?

Interest on a private purpose municipal bond On the exam, whenever you see a private purpose municipal bond, the interest on the bond is a tax preference item for the purpose of the alternative minimum tax.

If a gift tax is due, it is paid by the donor and based on the gift's value on the date it is given.

Issuers of municipal or municipal fund securities are exempt issuers and are not regulated or under the guidance of the MSRB or any other self-regulatory organization.

John is the annuitant in a variable plan, and Sue is the beneficiary. Upon John's death during the accumulation period, Sue takes a lump-sum payment. What is her total tax liability?

It is the proceeds minus John's cost basis taxed as ordinary income at Sue's tax rate. Annuity death benefits are generally paid in a lump sum. The beneficiary is taxed at ordinary income rates during the year the lump sum is received. The amount taxed is the amount of the lump-sum payment minus the deceased's cost basis in the investment.

It would be most unusual to see which of the following issued at a discount?

Jumbo CD Jumbo (negotiable) CDs are one of the few money market instruments issued at face value. Unlike those issued at a discount, they are interest bearing. Tbill, banker's acceptance, and commercial paper are all issued at a discount

The holder of a variable annuity receives the largest monthly payments under which of the following payout options?

Life annuity has the largest payout because less risk is assumed by the insurance company; there is no beneficiary in the event the annuitant dies.

Several investors open an account in joint tenancy. Which of the following statements regarding the account is true?

Mail need only be sent to one of the parties to the account. While mail only needs to be sent to one of the parties to the account, checks for disbursements from the account must be made payable to all parties and endorsed by all parties in order to be deposited. Any required forms pertinent to the account, such as a margin agreement or options agreement, must be signed by all parties.

Reverse churning

Most commonly seen in fee-based account

Moody's Investment Grade (MIG) ratings are applied to

Muni notes - Moody's Investment Grade ratings are applied to municipal notes, which are short-term municipal debts such as bond anticipation notes (BANs).

All of the following securities would be suitable investments for a traditional IRA except

Municipal bonds, which generate tax-free interest income, are unsuitable for retirement plans. One loses the federally tax-free income at distribution.

If an investor expects to have a large amount of passive income over the next two years, which of the following programs listed will most likely lead to the largest amount of shelter?

Oil and gas drilling Passive income can only be sheltered by passive loss, so the real estate income program will only add to the income. Oil and gas drilling programs allocate the majority of investment dollars to drilling. These are intangible drilling costs (IDCs), which are 100% deductible when drilling occurs. Undeveloped land has very little in the way of losses, and equipment leasing programs usually generate income shortly after starting.

Which of the following items appears on the confirmation statement for a when-issued trade of municipal bonds?

Principal or agency trade The capacity of the firm, principal or agent, must be disclosed on all confirms. The settlement date, accrued interest, and total price would not appear on a when-issued confirm.

All of the following risks are considered diversifiable except

Purchasing power risk, also known as inflation risk, is a systematic risk and, as such, is one that cannot generally be lessened through diversification. The other choices are forms of unsystematic (nonsystematic) risk and can be reduced through diversification.

A couple's home has an assessed value of $40,000 and a market value of $100,000. What will the tax be if a rate of 5 mills is used?

Real property tax is based on the assessed value assigned to the property by the municipality's tax assessor (in this case, $40,000). Property tax rates use the mill as a base unit. One mill equals $1 of tax per year for each $1,000 of assessed value. Five mills would equal $5 for each $1,000 of assessed value. Because there are 40 thousands, 40 × $5 equals $200 in annual tax. A shortcut method is as follows: take the assessed value, remove the last three 0s, and multiply by the number of mills of tax ($40 × 5 mills = $200).

Fairweather Securities Corp. (FSC), a registered broker-dealer, has invited several IARs from Econometric Advisory Services (ESA), a registered invested adviser that directs transactional business to FSC, to a seminar featuring a disquisition on current economic trends being presented by a leading economist. It would be permitted for FSC to cover which of the following expenses?

Registration fees for the seminar Payment for seminar fees, but not travel and transportation expenses, is permitted under the safe harbor provisions in Section 28(e) of the Securities Exchange Act of 1934.

Which of the following is not a type of corporate debt instrument?

Revenue bond. A revenue bond is a type of municipal bond.

An incorporated business model that allows flow-through of business income and losses directly to shareholders in order to avoid double taxation is

S corp The S corporation, the general partnership, and the limited partnership are business models where all income or loss flows through to the owners. This avoids the double taxation on the business level and owner level, as is the case with the C corporation. With C corporations, corporate earnings taxed once at the business level and again when they are paid out to shareholders as dividends. Because the question is asking about the incorporated business model, the correct choice is the S corporation.

Which of the following statements regarding SMA balances is true?

SMA balances may be withdrawn provided the withdrawal does not bring the account below minimum maintenance.

In a volatile market, which of the following option strategies carries the most risk?

Short straddle To establish a short straddle, the investor sells a call and a put; the short call carries unlimited loss potential.

On a single day, a customer purchases 15 TPL Sep 50 puts at 6 and 15 TPL Sep 50 calls at 1. If the price of TPL is $45 per share and the customer has no other security positions, what is this position called?

Straddle A long straddle is the purchase of a call and a put on the same stock with the same strike price and expiration. A straddle differs from a combination in that the strike prices and/or the expiration dates on a combination are different. A spread is a long put and a short put or a long call and a short call, rather than a put and a call.

Under the Uniform Practice Code, regular way transactions for common stock settle on

T+2

The owner of an IRA, age 45, has contributed $10,000 into the account and the IRA is now worth $20,000. The owner is going to convert the entire $20,000 into a Roth IRA. What are the tax consequences of this conversion?

The $20,000 is taxable as ordinary income in the year of the conversion.

The market-wide circuit breaker (MWCB) rule uses which of the following as the pricing reference point to measure a market decline?

The S&P 500 index recalculated daily The MWCB rule uses the S&P 500 recalculated daily as the pricing reference point to measure market declines for the purpose of triggering market circuit breakers to halt trading.

You are reviewing an investor's balance sheet. Which of the following items would be found on a balance sheet and help you determine the client's net worth?

The balance sheet reflects a person's net worth by comparing assets and liabilities. A 401(k) balance is an asset and credit card debt is a liability. Income and monthly bills, such as the electric bill, are found on the income statement.

A bond was issued 3 years ago with a coupon of 6%. The bond matures in 21 years and is callable at 103. Current market interest rates are 8%. Which of the following is most likely true?

The bond is selling at a discount.

All of the following would be considered when evaluating a municipal revenue bond's creditworthiness except

The collection ratio shows the percentage of property taxes that are collected. This would be relevant in evaluating general obligation bonds, which are backed by the taxing authority of the issuer. Revenue bonds, however, are backed by user fees, not taxes.

Who has the final responsibility for debt service on an industrial revenue bond?

The corporation leasing the facility The issuer of industrial revenue bonds is a municipality or an authority established by a municipality. However, no municipal assets or general revenues are pledged to secure the issue. The net lease payments by the corporate user of the facility are the source of revenue for debt service. Therefore, the ultimate responsibility for the payment of the principal and interest on an industrial revenue bond rests with the corporate lessee.

A registered representative of a FINRA member broker-dealer is gathering information from a prospective customer. When the representative uses the information to prepare a financial profile, which of the following would not be included?

The financial profile includes items with numbers. While risk tolerance is one of the most important aspects of information gathering, it cannot be quantified in the manner that debts, cash value, and IRA accounts are.

When a corporation issues a debt security, the terms of the loan are expressed in a document known as the bond's deed of trust. The deed of trust is sometimes referred to as

The indenture, sometimes also referred to as the deed of trust, states the issuer's obligation to pay back a specific amount of money on a specific date. A debenture is a debt security containing an indenture. Bond resolution is a term used for municipal bonds, not corporate debt.

While acting in a financial advisory capacity to a municipal issuer, a municipal securities dealer wants to be part of a syndicate in the underwriting of one of the issuer's new bonds. Which of the following statements regarding this situation is true?

This is recognized by the MSRB as a potential conflict of interest; municipal rules generally prohibit a broker-dealer from acting in both capacities.

Your client lives in a state with a personal income tax. To minimize that tax liability, it would probably be best for this client to purchase

U.S. Treasury bonds. With few exceptions, securities issued by the U.S. Treasury are the only government securities carrying an exemption from state income tax.

Complying with the suitability rules involves evaluating all of the following

Under FINRA Rule 2111, there are three main obligations: reasonable-basis suitability, customer-specific suitability, and quantitative suitability. There is no such thing as qualitative suitability.

Ratio call writing exposes an options investor to which of these?

Unlimited loss Limited gain Ratio call writers assume unlimited loss potential in a rising market and limited gain potential in a falling market. Maximum gain—while limited—occurs if the stock is trading at the strike price of the short calls at expiration.

Which of the following best describes a debenture?

Unsecured corporate debt

Voter approval may be required for new bond issues for construction of which of the following?

Voter approval may be required for new issues of GO bonds. State prisons and public high schools are among the facilities for the public good that are built and supported by GO issues.

What is the profit to a syndicate member if a syndicate is offering an 8½% bond at 100, the syndicate manager is giving a 0.75 concession and a one-point total takedown, and the syndicate member sells 1,000 bonds?

When a member of the syndicate sells a bond, they are entitled to the total takedown—in this case, one point ($10) per bond (1,000 bonds sold × $10 per bond = $10,000 profit). Remember that the concession would only go to those who are not members of the syndicate but are part of the selling group instead.

Which of the following would be considered a bearish strategy?

Writing a call Those who write call options benefit when the price of the underlying asset declines (bearish). It is just the opposite for those who write a put. Spreads are bearish when the low strike price is sold and the high strike price is bought. That results in a debit when it is a put spread and a credit when it is a call spread. Credit put spreads and debit call spreads are bullish because it is the low strike that is purchased and the high strike that is sold.

Yield-based options expire on

Yield-based options expire like stock options—on the third Friday of the expiration month, which is the last day of trading.

A customer writes two ABC Jul 15 puts at 2 when ABC is 14. If the contracts are closed at a premium of 4 when ABC is 13, the customer has

a $400 loss. The investor receives $400 in premiums (2 × $200) and pays $800 to close out the options (2 × $400), resulting in a net loss of $400 ($800 − $400).

A quotation on a municipal security between dealers is assumed to be

a bona fide quote/firm

In a variable life annuity with 10-year period certain, a contract holder receives

a minimum of 10 years of variable payments, followed by additional variable payments for life. The owner of a life annuity with 10-year period certain will receive payments for life, subject to a minimum of 10 years. If the contract holder dies before the period expires, the remaining payments are made to the beneficiary. An example would be if a contract holder of a life annuity with 10-year period certain died after 5 years, payments would continue for 5 more years to the beneficiary and then stop.

One of the ways in which U.S. government agency issues differ from those offered directly by the U.S. Treasury is that

agency issues typically carry higher returns than Treasury issues because of the lack of direct government backing. Agencies, with very few exceptions (GNMA being one), do not carry the direct backing of the U.S. Treasury.

AMLY common stock is traded on the Nasdaq Stock Market. The company is planning a major expansion and is going to fund it through the sale of 5 million shares of authorized but unissued stock. This would be

an additional primary offering (APO). The sale of stock that has never been issued represents the sale of new shares. New shares are always a primary offering. In this case, because AMLY already has publicly offered shares trading, these new shares would be an additional primary offering, (APO).

Under FINRA rules, customers who are approved to trade options must receive a copy of the OCC Options Disclosure Document

at the time of or before account approval. All customers who are approved by the ROP to trade options must receive a copy of the OCC Options Disclosure Document at or before the time the account is approved to trade options. It is the options account agreement that must be returned by the customer within 15 days.

When evaluating a client's suitability, which of the following would be considered a nonfinancial consideration?

attitude

growing problem of senior exploitation

authorized to contact the trusted contact person provided on the new account form by the customer and to disclose information about the customer's account in addressing possible exploitation

Regulation T controls the extension of credit from

broker-dealer to customer

If a U.S. corporation exports machine tools to Switzerland and will be paid in Swiss francs (SF), to protect against foreign-exchange risk, it should

buy SF puts Referring to U.S. based companies, Exporters buy Puts and Importers buy Calls.

The practice of dollar cost averaging requires the investor to

buy a security in a falling market and buy it in a rising market. Dollar cost averaging requires the investor to invest a fixed amount of money on a regular basis, regardless of whether the stock market is rising or falling.

On the order book, all of the following orders are reduced on the ex-date for a cash dividend except

buy stop. Only orders placed below the market price are reduced for cash dividends on the order book. Buy limits and sell stops are entered below the market price. Buy stops are entered above the market price.

Which of the following orders would be executed in a rising market? Buy stops Buy limits Sell limits Sell stops

buy stops and sell limits Buy limits and sell stops are entered below the current market and would be executed if the market were falling. Sell limits and buy stops are entered above the current market and would be executed if the market were rising.

A technology fund manager concerned about a downturn in the value of his portfolio would hedge by

buying narrow-based index puts. The portfolio consists of sector-specific securities, so broad-based index puts such as the OEX would not be appropriate. Instead, the manager should buy narrow-based index puts (for example, indices on technology and electronics).

A married couple who files jointly has a $5,000 long-term capital loss with no offsetting capital gains. Regarding the tax treatment of this loss, all of the following statements are true except

capital losses can be used to offset capital gains only. Capital losses are deducted from ordinary income, and therefore, reduce tax liability. The maximum that individuals or married couples can deduct is $3,000 annually. If the long-term capital loss exceeds the maximum, the excess is carried forward to future years until the loss is exhausted. Under current IRS regulations, $1 in losses results in $1 in deductions.

A method of analyzing limited partnerships by identifying the sources of revenues and expenses is known as

cash flow analysis.

A FINRA member firm sends a promotional piece to 23 individuals over a three-day period. Ten of these individuals are current customers of the firm. The other 13 are prospects whose names came from a commercially available mailing list service. Under the FINRA rule on communications with the public, this promotional piece would be considered

correspondence. Correspondence is defined as any written (including electronic) communication that is distributed or made available to 25 or fewer retail investors within any 30 calendar-day period. If it goes to more than 25, it is retail communication. The fact that some of the recipients are customers and others are prospects is irrelevant. It is simply the number of people during the 30-day period.

deferred compensation plan risk

deferred compensation plans may be somewhat risky because the employee covered by the plan becomes a general creditor if the business fails

FINRA Rule 2330 deals with a member's responsibility in the sale of certain insurance company-based products. Specifically the concern is with

deferred variable annuities. The subject of the FINRA rule is deferred variable annuities. It applies to the sale or exchange of this specific product.

Under FINRA rules, the maximum compensation that can be taken by sponsors selling direct participation programs is 10%. Up-front costs, such as commissions taken and accounting costs, do not reduce the beginning cost basis.

depletion. Depletion only applies to natural resources, such as oil or gas. Land does not get "used up" as does oil, gas, or coal, for example.

Each of the following is affected by the sale of securities in a restricted margin account except

equity. When securities in a margin account are sold, the market value in the account will decline, the debit balance will be reduced by the cash proceeds, and SMA will increase by 50% of the sale. Equity in the account will not be affected unless the customer decides to withdraw some of the proceeds through the use of SMA.

After receiving and accepting securities from another firm, a broker-dealer discovers that the securities received were not in good deliverable form. Recourse for the broker-dealer is to

file a reclamation. After receiving the securities, the broker-dealer can file a reclamation and demand good delivery.

Using a Rule 415 shelf registration, an issuer registered 40 million shares of common stock with the SEC. The registration is effective

for the following two years unless the issuer qualifies as a well-known seasoned issuer (WKSI), where it is three years.

An angry customer has written a letter of complaint and sent it to the registered representative handling the account. The first step the registered representative must take is to

forward the complaint to the proper supervisory person. Handling of complaints is a serious matter in this industry. Normally, the complaint is sent to the firm, who formally accepts the complaint and records the action taken. In this case, where the individual servicing the account receives the complaint, the firm's procedures manual will invariably require that the complaint be forwarded and specify to whom it should be forwarded. This is not something to be dealt with by the registered representative, at least not initially, and we don't have enough information to determine if it is the kind of complaint (e.g., forgery, theft) that must be reported to the firm's DEA.

As a new registered representative, there is much industry jargon to learn. If you overheard your manager discussing a stock power, it would be in reference to

good delivery of a stock certificate. A stock power is a separate document attached to a stock certificate (bond power for bonds) that is used instead of signing the actual certificate. A signed stock certificate is a negotiable instrument (like a blank signed check). If it is misplaced, lost, or stolen, anyone holding it can sell it and make delivery. Because the certificate itself is unsigned, good delivery requires both documents.

A company's balance sheet dated December 31 shows retained earnings of $100,000. You can deduce from this information that the company

has had $100,000 in undistributed profits since its inception Retained earnings represent the accumulated total of all earnings that have been retained in the company since its inception. It is quite possible that the company did not earn $100,000 in that particular year and does not have $100,000 in cash.

An investor acquires limited partner status in a direct participation program when

he and the general partner have both signed the subscription agreement The investor must sign a copy of the subscription agreement, but he is not considered a limited partner until the agreement is also signed by the general partner indicating acceptance of the limited partner.

An individual younger than age 70½ may contribute to a traditional IRA

if she has earned income. Any individual under age 70½ with earned income may contribute to a traditional IRA, even minors.

A covered call could be written to

improve the return on a portfolio. Writing a call will not necessarily lock in the profit. In the form of increased cash flow, it will improve the return on the portfolio.

If the value of securities held in a fund's portfolio increases, and the amount of liabilities stays the same, the fund's net assets

increase An appreciation in value of fund assets, without a corresponding increase in liabilities, leads to an increase in the fund's net asset value (total assets minus liabilities equals net assets).

An investor whose primary objective is a steady income flow would probably be best served by building a portfolio of

investment-grade debentures.

FINRA Rule 4530 states that each member shall promptly report to FINRA, but in any event not later than 30 calendar days, after the member knows or should have known of the existence of any of the following, otherwise than when the member or an associated person of the member

is the subject of an adjudication of bankruptcy by any domestic, military, or foreign court. FINRA Rule 4530 has very broad reporting requirements, but a broker-dealer or associated person being declared bankrupt is not one of them. If a member firm were to go bankrupt, FINRA would be notified immediately, not within 30 days.

Compared to U.S. government agency-backed CMOs, CDOs have

less prepayment risk Although there is some prepayment risk with CDOs, it is minimal when compared to CMOs. Unlike mortgages, which are frequently paid off early when homeowners move, those who move can take their cars or their credit cards with them and continue to make the payments. The same is true with refinancing. You don't see ads for people to refinance their auto loan to the extent you do with home mortgages. In generally, CDOs carry greater credit risk than CMOs backed by FHA and VA loans. The nature of CDOs, with the enormous variety of collateral options available, means that the liquidity of separate offerings tends to be less than that of these CMOs. Both of these are complex securities, but the experts in the field consider CMOs to be somewhat more so. The credit quality of government insured mortgages is more secure than that of credit card debt or automobile loans.

A new client of yours indicates that they remember hearing stories from grandparents who lived through the Great Depression of the 1930s. Those relatives lost almost everything they had in the stock market, and the client is not interested in seeing a repeat of the family history. When doing your information gathering, this would be an indication of the client's

level of risk tolerance

When opening an options trading account, a broker-dealer is required by FINRA to

make sure the options agreement is signed and returned to the firm within 15 days of the account being approved. If it is not, then the only options activity permitted is closing transactions. A Registered Options Principal (Series 4) or Sales Principal (Series 10) signs off on the account approval.

A mutual fund that charges 12b-1 fees may use the money to cover all of the following except

management fees 12b-1 fees may not be used to pay for the portfolio manager's fees, only for sales promotions and fees and other activities relating to the distribution of the fund's shares.

Sell order tickets must be

marked as either long or short.

High-tax-bracket investors are likely to receive the most favorable tax treatment from investing in

municipal bonds issued by a political subdivision of their state. Municipal bonds issued by a political subdivision of the investor's state are free of federal, state, and local income taxes. Treasury securities are taxed on the federal level, but not the state and local levels. GNMAs are taxed at every level. Qualifying dividends on preferred stock are taxed on all levels, although generally at a rate not exceeding 20%.

When investing in securities, there are many potential risks. When recommending a specific security to clients, a member firm

must disclose the existence of a control relationship between the firm and the subject company. While it is not often thought of as a risk, disclosing the existence of a control relationship between the member firm and the subject security is something that must be done. There is no question that this is material. Not all recommendations are for securities that will outperform the market. What about a stock recommendation to sell short? Buyer beware (caveat emptor) has no place in this industry.

An investor in an oil and gas limited partnership program is subject to the economic consequences of all of the following except

nonrecourse loans. Nonrecourse loans only have economic consequences for investors in real estate programs.

Advertisements for the Abstemious Balanced Fund (ABF) describe the investment as a no-load fund. In order to make this claim, the fund must

not have a conditional deferred sales charge When a fund promotes itself as a no-load fund, not only must there be no front-end load, there cannot be a back-end load (CDSC) either. The 12b-1 charge maximum is 0.25%. The concept of breakpoints applies solely to Class A shares (front-end load).

Your clients, an elderly retired couple on a small fixed monthly income, want to write uncovered (naked) calls in their joint account to generate income. For this account, this option strategy would most likely be deemed

not suitable, because it is a speculative strategy with unlimited loss potential.

Your client wishes to invest $50,000 into shares of the ACE Mutual Fund. This morning's financial news indicated that the POP for ACE was $10.86, while the NAV was $10 per share. The client's order is placed at 2:00 pm Eastern time. Based on this information, you could confirm to the client a purchase of

nothing yet, as you must wait for the POP to be computed based on the day's close. Mutual funds use forward pricing, so we never know what we'll be paying per share (if purchasing) or receiving per share (if redeeming) until the next calculated price.

An arbitration case dealing with a dispute between a customer and a member firm in the amount of $72,000 will be heard by

one arbitrator, unless both parties agree to three. When the amount in dispute is more than $50,000 and no more than $100,000, the usual number of arbitrators is one. If both parties agree, it can be three. When the dispute involves an amount greater than $100,000, there are three arbitrators unless both parties agree to one. In a dispute with a customer that involves an amount greater than $100,000, the customer can request that all three arbitrators selected be from the public sector.

Bob Smith, who is in his 40s, has just become covered by an extremely generous defined benefit retirement plan at his company. He has decided he no longer needs his variable annuity for retirement purposes and wants to use the money for a trip to Africa. Over the past 10 years, he has invested $60,000 in the annuity, and its net value is now $80,000. If Bob should go ahead and surrender the annuity, the tax consequences will be

ordinary income tax on $20,000 and a $2,000 penalty. If an annuity is cashed in, the growth and accumulation portion of its value ($20,000 in this case) is taxable as ordinary income. If the annuitant is under the age of 59½, he must also pay a 10% penalty on the growth withdrawn, a penalty of $2,000 in this case.

One way in which open-end investment companies differ from closed-end investment companies is that an open-end investment company's shares

outstanding will vary in number at any point in time. Open-end investment companies are capitalized with a continuous offering of new shares. As a result, the number of shares outstanding is constantly changing. It is the closed-end company, traded in the secondary markets, whose share prices are based on supply and demand, which causes them to be bought or sold at a premium or discount to the NAV.

All of the following are common to both DPPs and REITs except

pass-through of losses. Both DPPs and REITs are professionally managed pools that pass through income and capital gains distributions to participants. REITs, unlike DPPs, do not pass through losses.

All of the following will affect the working capital of a corporation except

payment of a cash dividend. Working capital is defined as current assets minus current liabilities. On the declaration date, the future dividend payment is "booked" as a current liability (dividend payable). When the payment date comes, disbursement of the cash dividend will reduce current assets (cash) and current liabilities (dividend payable) by the same amount, leaving working capital unchanged.

An order memorandum or ticket must be prepared

prior to order execution Order tickets must be prepared prior to order execution. Please read the question carefully. Preparation is the first step and that must be done before the order can be sent for execution. After the execution report, the order ticket is completed with the execution information.

The purchaser of a general obligation (GO) municipal bond should be concerned with property tax assessments. the maintenance covenant. market risk. feasibility studies.

property tax assessments and market risk

All of the following terms are associated with general obligation (GO) bonds except

protective covenants. The protective covenants are found in the trust indenture of a revenue bond. Among the more common protective covenants are the rate covenant and the maintenance covenant. The former is a promise to maintain rates sufficient to pay expenses and debt service.

When reviewing a client's account, your supervisor notices that although each recommendation appears to be suitable based on that client's profile, there is a concern regarding the frequency of activity in the account. This is an example of

quantitative suitability

Programs allowing for the direct pass-through of losses and income to investors include all of the following except

real estate investment trusts (REITs). REITs allow for the direct pass-through of income, but not losses. The other choices are forms of business that allow for pass-through of income and losses.

An investor who purchases 20-year Aaa rated corporate zero-coupon bonds would be least concerned with

reinvestment risk. Zero-coupon securities have no reinvestment risk because there are no interest payments to reinvest. All fixed-income securities have purchasing power (inflation risk), especially when the maturity is as long as 20 years. The same is true for interest rate risk. In fact, zero-coupon bonds, because of their long duration, are the most sensitive to changes in interest rates. Is there default risk? Yes, even with a triple A rating. A lot can happen in 20 years.

You have a client who plans to liquidate some CDL stock to help pay for an upcoming family vacation in late August. When checking the account record, you find the following transactions: Jan 4, 100 shares @ $43 Feb 8 100 shares @ $39 May 11, 200 shares @$48 The client needs about $4,000 and the CDL is currently selling for $44 per share on July 31. From a tax standpoint, you should probably recommend that the client

sell 100 of the shares purchased on May 11. By using the share identification basis and selling the shares purchased in May at $48 per share, the client realizes a loss of $400. Selling either of the others results in a short-term capital gain, taxed at ordinary income rates (all transactions are less than a 12-month holding period). Don't get hung up on the fact that the investor will receive $400 more than needed (100 shares @ $44 = $4,400); the question is looking for tax considerations.

A customer long 100 shares of XYZ stock who wishes to reduce risk and generate income should

sell an XYZ call If the customer sells a call, the risk of owning the stock is reduced by the call's premium. Receipt of the premium satisfies the customer's income objective.

Covered put writing is a strategy where an investor

sells a put on a stock he has sold short. The customer sells the put to generate income. The short stock position provides the necessary cash should his short put be exercised, forcing him to buy the stock.

When processing the morning mail, your firm's trading department notices a confirmation for an unrecognized trade. The proper procedure to follow is

sending a DK notice to the sender of the confirmation. DK stands for don't know, and the DK notice is used when one member to a trade does not recognize that transaction. Reclamation is returning or demanding the return of securities previously delivered. Rejection is done at the time of delivery.

A client with an options account contacts the registered representative handling the account with instructions to open the following spread: Buy 1 ABC 100 call and Sell 1 ABC 105 call at a 5-point debit. Under FINRA rules, this order

should be refused. The order should be refused because it is impossible for it to be profitable. This is a bull call spread (but that is not the correct answer here because it has nothing to do with FINRA rules) and will become profitable when the spread widens. With strike prices of 100 and 105, it can never widen more than 5 points. If the client paid 5 points for the spread, once commissions are factored in, the client must lose money and certainly cannot profit. FINRA looks at this as an uneconomic position, and the firm should refuse to take the order.

If a customer of a member firm would like to file a claim under the Code of Arbitration Procedure, the claim will not be eligible once

six years have elapsed from the occurrence or event giving rise to the claim.

Determining a client's investment objectives is an important function of being a registered representative. A customer who identifies as having a conservative investment posture would probably avoid

speculation

One respect in which TIPS bonds differ from all other U.S. Treasury securities issued at par value is that they

subject the investor to phantom income. The inflation protection of a TIPS bond comes from the semiannual adjustment to the principal value. Those increases are reported to the IRS as ordinary income to the investor. It is called phantom income because the investor does not "see" that money currently, but still must pay taxes on it. Like other Treasuries issued at par (T-notes and T-bonds), interest is paid semiannually at the fixed coupon rate. The actual interest will vary based on the principal adjustment, but the coupon is fixed. As with the other two mentioned, quotes are in 32nds.

Your customer is interested in buying call options on CDL common stock. The client asks you, "Who issues CDL options?" The proper response is

the Options Clearing Corporation. The issuer and guarantor of the options covered on the exam is the Options Clearing Corporation (OCC).

When a person is calculating cost basis for a nonqualified variable annuity, the person is using

the after-tax dollars contributed. For a nonqualified variable annuity, cost basis for the annuitant would use the after-tax dollars contributed.

Variable annuities generally include an assumed interest rate (AIR). This is

the annual rate of return required to maintain the level of annuity payments.

An investor owns a 6% bond issued by ABC Corporation that is callable at 102 ($1,020) next May 1. All of the following statements are true regarding the call except

the bond is probably being called by the issuer because interest rates went up. When interest rates fall, look for a call! If interest rates fall to, say, 4%, the issuer would want to get rid of the 6% debt and perhaps issue a new bond at the 4% rate. Up until the call date, the investor has call protection; as of the call date and every date thereafter, the investor has call risk.

Opening a margin account involves a number of different documents. The document describing how the interest on the margin debt is calculated is generally known as

the credit agreement

All of the following statements regarding industrial revenue bonds (IRBs) are true except

the credit rating of the bonds is dependent on the credit rating of the municipality. The debt service for IRBs is derived from the lease payments made by the leasing corporation to the issuing municipality. Therefore, the credit rating of the bonds is dependent on the credit worthiness of the leasing corporation, not the issuing municipality.

A brokerage order ticket must contain all the following except

the date the account was opened. The date of the trade will absolutely be on the order ticket. However, the date the account was opened will not be included.

When creating a diversified municipal bond portfolio, all of the following should be considered except

the denomination of the bonds included in the portfolio.

When a customer, who is at least 59½, withdraws money from a traditional IRA that has been funded totally with deductible contributions,

the entire amount withdrawn is subject to taxation at ordinary income tax rates.

If a prospectus for a variable life insurance product contains hypothetical projections of returns,

the maximum return permitted is 12%, and there must be an illustration showing a 0% return as well. In discussions of variable life insurance with customers, projections of hypothetical returns may be used. This is to demonstrate how the program really works and the risks involved if the projections are not met. The maximum allowable hypothetical rate of return is 12%, and there must also be a projection shown assuming a 0% return. Variable life insurance must not be sold as an investment product. It must always be emphasized that the policy should be purchased to meet an insurance need.

The STU Corporation has issued common stock, preferred stock, promissory notes, and mortgage bonds. Should STU enter bankruptcy proceedings, the order of payment against claims would be

the mortgage bonds, the promissory notes, the preferred stock, and the common stock. In a bankruptcy, secured creditors, such as those with a mortgage against real property, have the first priority. They are followed by unsecured creditors, such as holders of promissory notes, with stock holders coming last. Preferred stock is "preferred" over common stock in both liquidation priority and payment of dividends.

A generic ad for an investment company placed by a broker-dealer would contain

the name of the broker-dealer, but not the name of the investment company. Generic advertising of investment companies presents a nonspecific introduction to investment company shares. A specific fund or investment company is not mentioned in generic advertising, but the broker-dealer who is placing the ad must be named.

An investor purchases a bond on its initial public offering. Even though the bond has a maturity value of $1,000 in 10 years, the offering price is only $600. If the bond is held to maturity,

there are no tax consequences to report. A bond issued at a significant discount from its maturity value is known as an original issue discount bond (OID). In the case of a corporate bond, the computation is more complex than can be tested, but there are two things you need to know: A portion of the discount is taxed as ordinary income each year until maturity, even though it is not actually received. This is called phantom income. Each year's taxable amount is reported on Form 1099-OID. Because a portion of the discount has been taxed each year, at maturity there are no tax consequences—no gain, no interest.

There are many different types of asset-backed securities, but the common theme uniting all of them is

they are supported by a contractual obligation to pay. Asset-backed securities (ABS) are structured debt financing backed by a contractual agreement to pay. They are "first cousins" to MBS (mortgage-backed securities), with the primary difference being that the collateral is not real estate. These securities trade in the OTC market. One of the benefits of the structured package is that the investor is not relying on payments from a single borrower; rather, there is a pool of loans, whether they be auto, credit card, or others.

You have a client who invested in the PQR Growth Fund 10 years ago and now, as retirement age approaches, asks you about using the exchange privilege to move into the PQR Balanced Fund. The client should know that

this exchange is considered a taxable event as of the date of the exchange. The exchange privilege allows for an exchange at net asset value (NAV) between funds that are members of the same "family." The exchange is considered a taxable event. Because the exchange is made at NAV, the concept of breakpoint is irrelevant.

Which of the following is the largest component of a municipal underwriting spread?

total takedown The underwriting spread is the entire amount. Total takedown is made up of the additional takedown and the concession, which makes it the largest component. The management fee is the smallest.

Which of the following would be found on a when-, as-, and if-issued confirmation?

trade date and price Information that does not appear on a when-issued confirmation can easily be remembered as SAT (settlement date, accrued interest, and total amount due). The trade date and price per bond are included on the when-issued confirmation.

Under Regulation T, when a customer purchases securities, payment must be received by the broker-dealer no later than

two business days after the settlement date.

Common risk factors found when investing in penny stocks would include all of the following except

unfavorable tax treatment.

In a direct participation program (DPP) limited partnership, the general partner has

unlimited liability and an active role. In a DPP limited partnership, the general partner is the active partner managing the business who assumes unlimited liability. Limited partners who take an active role jeopardize their limited liability status.

A client places a sell stop order good for the day only. Under NYSE rules, you must

write an order ticket upon receipt of the order. Order tickets must be written and time stamped upon receipt of the order.

A commonly used investment to provide a defined sum in the future, such as for college education or retirement, is

zero-coupon bonds. The key to this question is the defined sum. Zero-coupon bonds are usually purchased at a deep discount, which helps a small initial deposit grow into a substantial sum at maturity.

An investor sold 100 shares of RAN common stock short at $50 per share. The RAN is now at $38. The investor is still bearish on the stock but would like to protect that gain. What would you recommend if this were your client?

Enter a buy stop order at $40 Buy stop orders are used as a protective tool for short sellers. In this case, if the stock should begin to rise from its current price of $38, once it reaches or exceeds $40, a buy order at the market is entered. The stock purchased is used to cover the short position, and the investor's profit is the $50 sale price minus the cost of the purchase. Buy stops are always placed above the current market. Because the investor is short, the only protective order would be a buy, not a sell (you buy protection)..

For each violation of FINRA or MSRB rules, FINRA may impose which of the following sanctions?

Fines, censure, expulsion. Restitution is not one of the sanctions that may be imposed by FINRA. However, that could be imposed by a court.

In a new margin account, a customer sells short $60,000 worth of ABC stock and deposits $30,000 to meet the Regulation T requirement. If the value of ABC falls to $55,000, the special memorandum account (SMA) balance in the account would be

For every $1 decrease in market value in a short account, $1.50 of SMA is created. Therefore, if the market value falls by $5,000, the SMA balance would be $7,500.

A customer has been following several investment company quotes in the newspaper. She notices that the GEM Fund has a net asset value (NAV) of $12 and a public offering price (POP) of $12.50, and that the ABC Fund has an NAV of $11.50 and a POP of $10.98. The customer should conclude that

GEM may be an open- or closed-end fund, and ABC is a closed-end fund. The price for open-end funds is determined by adding the sales charge to the NAV. An open-end fund can never have a POP less than its NAV; therefore, ABC cannot be an open-end fund.

Which of the following statements regarding the Government National Mortgage Association (GNMA) is true?

GNMA is a government-owned corporation that approves private lending institutions, such as banks and mortgage companies, to originate eligible loans, pool them into securities, and sell the GNMA mortgage-backed securities to investors. GNMA does not originate loans, and it does not issue or sell securities.

Which of the following securities has the direct backing of the U.S. Treasury?

GNMA's Securities issued by the Government National Mortgage Association (GNMA) are the only agency security with the direct backing of the U.S. Treasury. Other agencies are not directly backed. Treasury stock is a corporate issue and has nothing to do with the U.S. Treasury.

Under what circumstances would the fiduciary of a qualified corporate retirement plan be permitted to write covered calls on the securities in the portfolio?

If this strategy is consistent with the objectives of the plan As covered calls are not considered to be a speculative option strategy, they would be permitted as long as the strategy is deemed prudent and is consistent with the objectives of the plan. No outside approval is required.

A 65-year-old man called the branch manager to complain about a recent exchange of a deferred variable annuity proposed and performed by the registered representative handling his account. The customer said he was unaware that there would be charges associated with the transaction and was shocked that the account value diminished substantially during a recent downturn in the market. The manager should do which of the following?

Interview the registered representative to ascertain whether firm procedures were adhered to with regard to suitability and disclosure of charges and risks associated with exchanges. Member firms may not recommend to customers the purchase or exchange of a deferred variable annuity unless the associated person has made proper disclosure of the features of variable products, including surrender charges, tax penalties, features of riders, and insurance components, and obtained a reasonable basis to believe the transaction is suitable (see FINRA Rule 2330). Rules only apply to written complaints.

If a customer is in a low federal income tax bracket and his main investment objective is current income, which of the following securities should the agent recommend?

Investment-grade corporate bond To maximize income, the best recommendation of the choices listed is the corporate bond, which offers a higher yield than a government bond with a similar maturity.

The Investment Company Act of 1940 contains a number of terms used to describe investment companies. When used as an adjective, the term diversified would apply to which type of investment company?

Management company The Investment Company Act of 1940 divides investment companies into three principal classifications. Those are the face-amount certificate company, the UIT, and the management company. Management companies are further divided into open-end and closed-end companies. The act goes one step further and has management companies divided again into diversified and nondiversified companies. It is not expected that you will have to know what a BDC (business development company) is.

An investor opens the following options position: Write 1 RAN Dec 35 call @4; short 1 RAN Dec 30 put @1¼. What is the investor's maximum gain, maximum loss, and breakeven point?

Maximum gain is $525; maximum loss is unlimited; breakeven points are $24.75 and $40.25. he first step is to identify the position. This is a short combination; a short put and a short call with different terms. That means we are going to have two breakeven points. The maximum loss is unlimited because one of the positions is an uncovered call. The maximum profit is the premiums (credit) received of $525. Breakeven points follow the call-up and put-down rule. That is, add the premiums of $5¼ to the strike price of the call ($35 + $5.25= $40.25) and subtract the premiums of $5¼ from the strike price of the put ($30 ‒ $5.25 = $24.75).

Arbitration and mediation are two services provided by FINRA to settle disputes between members. Regarding these services, which of the following statements are not true?

Mediation is mandatory; arbitration is not and If arbitration is unsuccessful, the dispute moves on to mediation are FALSE. Arbitration is mandatory in disputes between members. If mediation takes place and is not successful, the dispute moves on to arbitration. The person who served as mediator may not be an arbitrator in the same dispute.

To comply with the regulations regarding customer identification programs, the minimum identifying information that must be obtained from each customer before opening an account includes which of the following?

Name and Tax ID

An investor purchased a single premium deferred variable annuity 20 years ago. The premium deposit was $50,000. The account is now worth $200,000 and the investor is still working. When does the investor have to begin taking required minimum distributions?

Never with a nonqualified annuity On the exam, unless stated to the contrary, every annuity is nonqualified. One of the benefits of nonqualified annuities is that there is no age at withdrawals must commence. In general, earnings withdrawn prior to age 59½ are subject to the additional 10% penalty on top of tax at ordinary rates.

Which of the following types of municipal bonds is subject to statutory debt limits?

Only GO bonds, which are backed by the taxing authority of the issuer, are subject to statutory debt limits.

Which of the following would be considered an equity security?

Preemptive rights Rights (and warrants) are included in the term equity security

The interest on which of the following municipal securities may be considered preference income for alternative minimum tax purposes?

Private purpose bonds Interest on private activity municipal bonds is included in the taxable income of an investor who is subject to the alternative minimum tax.

Which of the following statements concerning Section 529 plans are true?

Qualified withdrawals are exempt from federal income tax. Up to $10,000 per year can be used for K-12 tuition expenses. The withdrawals from Section 529 plans are federally tax exempt, but they may be taxed as income in some states. The money that is invested in a Section 529 plan is always after tax. To avoid taxation, the withdrawals must be used for qualified education expenses (e.g., tuition, books, lecture fees, lab fees), including up to $10,000 per year for K-12 education. Withdrawals to pay expenses that are not qualified incur tax liability.

An investor in an equipment-leasing direct participation program (DPP) using straight-line depreciation would probably not be concerned about

Recapture of deductions is a concern when accelerated, but not when straight-line depreciation is used. In any business, there is always concern about the quality of the management. By and large, DPPs are not liquid investments, so an investor needing a quick sale may have problems. The nature of DPPs tends to make them more sensitive to legislative risk than most other securities.

Sam Brown has several stock rights. Which of the following is not an alternative regarding these stock rights?

Redeeming them from the issuer for cash Rights are not redeemable by the issuer. They may be sold in the secondary market or given to someone else to trade or exercise. If exercised, rights are exchanged for an appropriate number of shares of the underlying common stock.

Which of the following securities is sold at auction?

T-bills, T-notes, and T-bonds are sold through auction. These auctions award securities to the most competitive bids. Agency securities are sold through selling groups appointed by the agency.

A customer gives you a limit order to buy 500 shares of XYZ at 30. You erroneously buy 1,000 shares at 29. The customer is entitled to

The customer is entitled to 500 shares at 29. A buy limit order may be executed at the limit price or better (lower). If the firm buys more shares than indicated on the order ticket, the customer cannot be held responsible.

All of the following have an impact on the marketability of a block of municipal bonds except

The dated date has no effect on marketability. A close call date or low call premium can make an issue less marketable because the chance of a call is greater. Maturity, quality, and the size of the block affect marketability.

FINRA Rule 2330, which deals with members' responsibilities regarding variable annuities, applies under which of the following circumstances?

The initial purchase of a deferred variable annuity. The initial subaccount allocations. When a variable contract is annuitized (distributed in regular payments, not as a lump sum), the number of accumulation units is multiplied by the unit value to arrive at the account's current value. An annuity factor is taken from the annuity table, which considers, for example, the investor's sex and age. This factor is used to establish the dollar amount of the first annuity payment. Future annuity payments will vary according to the separate account's performance.

The performance of the XYZ Growth Fund has been in the top 1% of all funds in its category for the past 1-, 5-, and 10-year periods. Which of the following would be the biggest risk factor to an investor investing in this fund?

The manager's tenure is six months Although one cannot predict the future from the past, when a portfolio manager has consistently been ranked at the top, it is not considered a major risk to bet on a winner. The problem here is that almost all of that performance was achieved under the direction of previous management. With only six months on the job, the new manager is untested and there is no way to know how the future performance will rank. You might see this referred to as tenure risk. Diversification is one of the benefits, not risks, of a mutual fund. In a growth fund, one does not expect a high dividend yield.

One of the computations in a margin account is that of the SMA. Which of the following actions would cause the SMA to increase?

The market value of short securities decreasing Excess equity creates SMA in the account. Excess equity is created when the stock in a margin account moves in a favorable direction. In the case of a short margin account, the equity goes up as the market value of the securities goes down. In the case of a long account, a decline in the market value causes the equity to decrease. The purchase of securities in a long account may decrease SMA if SMA is used to meet the margin call. If not, SMA remains the same; it does not increase. Withdrawing cash will cause the SMA to go down if the funds are withdrawn from the SMA. Otherwise, the SMA remains the same; it does not increase.

Which of the following statements regarding customer confirmations is not true?

They must disclose the current yield on equity securities. There are specific yield disclosures required for debt securities, but not for equities. The final dollar amount of the transaction always reflects any agency commissions. The firm's capactiy (agency or principal) is always disclosed. When the firm is a market maker, that information is specified as well. The date and time of the transaction (or the fact that this will be furnished upon written request) is a part of every confirmation.

It is important for a registered representative to be able to distinguish between a client's investment objectives and investment constraints. Which of the following is an example of an investment constraint?

Time horizon

If an investor establishes a call spread, and buys the lower exercise price and sells the higher exercise price at a net debit, he anticipates that

the spread will widen. Debit spreads are profitable when both sides are exercised or the spread widens between the premiums. Credit spreads are profitable when both sides expire or the spread narrows between the premiums.

A fundamental analyst researching a stock is concerned with all of the following except

the volume of shares traded. A fundamental analyst is concerned with the economic climate, the inflation rate, how an industry is performing, a company's historical earnings trends, how it is capitalized, and its product lines, management, and financial statement ratios, such as the P/E ratio. A technical analyst is concerned with trading volumes or market trends and prices.

Which of the following debt securities would be most likely to offer a conversion feature into common stock?

A debenture

Which of the following client statements describes an investment objective rather than an investment constraint?

"I want to maximize my income." income is an objective. Liquidity, tax considerations, and personal attitudes are investment constraints.

One of your customers, age 52, wishes to open an IRA. His annual income is more than $200,000 and consists entirely of income from rental real estate and income from a trust fund. What amount may your customer contribute to his IRA this year?

$0 To open an IRA, a person needs earned income. Income from rental real estate is passive income, while income from a trust fund is portfolio income. This customer has no earned income.

A company reported annual earnings of $2.40 per share and paid annual dividends of $0.60. If the dividends were distributed quarterly, what was the amount and payout rate?

$0.15 at 25% One-quarter of $0.60 is $0.15. $0.60 is 25% of $2.40.

When the inside market (best bid and best offer) for XYZ stock was 17.30-17.60, a market maker bought 100 shares from a customer at 16.90. At the time of the trade, the market maker's market was 17.25-17.70. What was the amount of the markdown?

$0.40 Markdown is always based on the inside quote. In this case, the inside bid is 17.30 and the difference between that and the 16.90 buying price represents a $0.40 markdown.

A customer has a margin account with a market value of $20,000, a debit balance of $12,000, no special memorandum account (SMA), and Regulation T at 50%. If the customer sells $2,000 worth of stock, the amount released to SMA is

$1,000 This is an example of a restricted account with equity below the 50% Regulation T requirement. In a restricted account, 50% of the sale proceeds is released to SMA (50% × $2,000 = $1,000).

An investor purchases a TIPS bond with a 3% coupon. During the first year, if the inflation rate is 8%, the principal value of the security at the end of that year will be closest to

$1,081.60. The principal value of a TIPS bond is adjusted semiannually by the inflation rate. The exact calculation would be $1,000 × 104% × 104%, which equals $1,081.60. Each six months, the interest is paid on that adjusted principal and that is why the security keeps pace with inflation. There is a shortcut that will always work on the exam. Just recognize that the principal value increases based on the inflation rate compounded semiannually. Take the simple interest rate and choose the next highest number. In this example, 8% simple interest would be $80 (which would always be one of the choices). Because the computation is done twice per year, the compounding effect makes the correct choice slightly higher.

ABC Corp. has outstanding a 10% noncumulative preferred stock. Two years ago, ABC omitted its preferred dividend. Last year, it paid a dividend of $5 per share. In order to pay a dividend to common shareholders, each preferred share must be paid a dividend of

$10 In order for a common dividend to be paid when the issuer has noncumulative preferred stock outstanding, the preferred dividend must be satisfied. In this case, the stated dividend is 10% or $10 (10% × par of $100).

A customer has the following margin account balance: Market value: $50,000 Balance: (DR) $26,000 SMA: $0 Regulation T: 50% If a customer sells securities in the amount of $20,000 in this account, what is the maximum amount she can withdraw from the account after the settlement date?

$10,000 When stock is sold in a restricted margin account, 50% of the proceeds can be withdrawn. This account is restricted because the equity of $24,000 (LMV of $50,000 − DR of $26,000) is less than Regulation T ($25,000 = 50% of the LMV) but more than minimum maintenance ($12,500 = 25% of the LMV).

A customer purchases ten 8% Treasury notes at 101-16. What is the dollar amount of this purchase?

$10,150 Though the denomination of the T-notes purchased is not given, always assume par ($1,000) unless told differently in the question. Remember that government notes and bonds are quoted in 32nds. Therefore, a quote of 101-16 means 101 plus 16/32. 101 plus 1/2 = $1,015; $1,015 × 10 bonds = $10,150. or 101.5 x100 = $10,015

An investor purchased 200 shares of DCAST common stock at $200 per share. What is the adjusted cost basis per share of this position after the company pays a 100% stock dividend?

$100 The total value of the initial position is unchanged, remaining at $40,000 (200 times $200). After the stock dividend, the investor owns 400 shares (200 times 100% = 200 + 200 = 400). Therefore, the adjusted cost basis is $100.00 per share ($40,000 divided by 400 = $100). Perhaps you recognized that a 100% stock dividend has the same effect as a 2:1 split. That is, the stock's cost basis is cut in half. It is important to remember that anytime there is a distribution resulting in additional shares (stock split, stock dividend), the cost basis per share is reduced while the total account value remains the same.

In a new margin account, a customer buys 300 shares of ABC at $40 per share, 100 shares of the Ajax Mutual Fund at $24, and 10 PDQ Aug 30 calls at 4. The customer will receive a margin call for

$12,400. The customer must pay 50% of the value of the stock and 100% of the value of the mutual fund shares and the options because these securities are nonmarginable and require full payment. To calculate the total payment required, add $6,000 (50% of $12,000) plus $2,400 (Ajax) plus $4,000 (PDQ calls) to arrive at $12,400.

A customer's long margin account contains the following securities: 100 shares of DEF, CMV $40 per share 200 shares of AMF, CMV $50 per share 100 shares of KLP, CMV $80 per share The current debit balance in the account is $10,800. If each of the securities held in the account were to appreciate by $5 per share, the equity in the account would be

$13,200. Before the $5 increase, the total long market value in the account is $22,000 and, therefore, the equity is $11,200 ($22,000 - $10,800 = $11,200). If each of the 400 shares in the account increases by $5, which represents an increase of $2,000 in long market value, the equity will increase by the same amount. After the increase, the long market value is $24,000 and the equity is $13,200 ($24,000 - $10,800 = $13,200).

If an M&N 1 corporate bond issued at par with a 6% coupon is later purchased in August for 97 plus accrued interest of $16, how much taxable interest must the investor report for the year?

$14 The purchaser of a bond pays the seller the interest that has accrued since the last interest payment date. A purchaser in August will pay the interest that has accrued since May 1. Then, on November 1, the investor will receive the entire six months of interest. We are told that the investor paid $16 in accrued interest. That is income to the seller. Then, when the November payment of $30 (6% coupon is $30 semiannually) is made, the investor must report the amount over the accrued interest paid out as income. In our question, that is $30 minus $16 = $14.

A customer's margin account shows a debit balance of $10,000. Federal law permits the broker-dealer to rehypothecate a maximum of

$14,000 of the customer's margin securities. When a customer buys securities on margin, the broker-dealer holds the purchased securities as collateral for the margin loan. That is known as hypothecation. In most cases, broker-dealers rehypothecate the securities to a lending institution to recover the funds they loaned to the margin client. SEC rules limit the rehypothecation to 140% of the customer's debit balance (140% × $10,000 = $14,000).

A customer opens a new margin account, and the first trade is the short sale of 100 shares of ABC, at a price of $18 per share. What is the required margin deposit?

$2,000 The minimum of $2,000 is never waived for short sale margin requirements. If the short sale calculation is less than $2,000 ($1,800 in this case), the customer is still required to deposit the $2,000 minimum.

If a customer has a restricted margin account with special memorandum account (SMA) of $2,500, how much must he deposit to purchase $10,000 worth of stock?

$2,500 The purchase of $10,000 requires a $5,000 deposit, which can be reduced dollar for dollar by the existing SMA.

A fundamental analysis is reviewing a corporation's income statement. For the period, the company reported net sales of $10 million, cost of goods sold of $6 million, depreciation expense of $1 million, interest on long-term debt of $1 million, and income taxes of $500,000. With this information, the analyst knows that the company's cash flow from operations was

$2.5 million. Cash flow from operations is the net income plus the depreciation. The net income is found after deducting all the expenses plus the income taxes from the net sales. Those expenses are $8 million ($6 million + $1 million + $1 million) resulting in pretax income of $2 million. After-tax income is $1.5 million. Add the $1 million depreciation to that to arrive at the answer of $2.5 million. Note that some accountants include the depreciation in COGS (cost of goods sold), but if that is not mentioned in the question, it is a separate expense.

A corporation has $20 million net income after taxes, 7 million common shares outstanding, and $15 million of 6% preferred stock ($25 par). What is the corporation's earnings per share (EPS)?

$2.73 Begin by calculating how much of the net income is available for common stockholders (net income after taxes minus preferred dividends results in the earnings available for common stockholders). The preferred stockholders received $900,000 in dividends ($15 million par × 6% = $900,000), or 600,000 shares × $1.50 per share = $900,000). After subtracting $900,000 from the net income of $20 million, this leaves $19.1 million (earnings available for common stockholders). Compute EPS (earnings available for common ÷ number of common shares outstanding = $19.1 million ÷ 7 million shares = $2.73 per share EPS).

A customer's margin account shows the following entries: MV = 25,000 DR = 15,000 You may inform the customer that he will receive a maintenance call if the LMV falls below

$20,000. To determine long market value at maintenance, divide the debit balance by 0.75 (the complement of the maintenance margin requirement of 0.25). $15,000 divided by 0.75 equals $20,000.

A customer has the following accounts: Market value: long account $35,000; short account $40,000 Balance: long account (DR) $23,000; short account (CR) $60,000 SMA: long account $3,000 Regulation T: 50% What is the combined minimum maintenance requirement for the long and short margin positions?

$20,750 The minimum maintenance for long accounts is 25% of the LMV (25% × $35,000 = $8,750). The minimum maintenance for short accounts is 30% of the SMV (30% × $40,000 = $12,000). In this case, $8,750 plus $12,000 equals a combined maintenance of $20,750.

A customer writes 1 XYZ Sep 45 put at 6 and 1 XYZ Sep 35 call at 6 when XYZ is at 40. Before expiration, if XYZ is at 43, and the customer closes her positions at intrinsic value, the customer has

$200 gain The customer collects $1,200 in premiums for writing the options (6 + 6), but later pays $200 (45 − 43) to close out the put and $800 to close out the call (43 − 35). In this case, $1,200 received minus $1,000 paid leaves a gain of $200.

GC, Inc., is proposing an additional public offering of common stock. It conducts a rights offering to its current shareholders at $55 per share, plus five rights. If the market price of GCI is $70 after the ex-rights date passes, what is the value of one right?

$3 Because the stock is selling ex (after ex-rights), the formula is ($70 − $55) / 5. ($70 − $55 = $15) ($15 / 5 = $3).

If a customer has a long margin account with a market value of $20,000, a debit balance of $12,000, and SMA of $5,000, how much cash can the customer withdraw from the account?

$3,000 SMA is a line of credit with one restriction: it may not be used if account equity would fall below minimum maintenance. In this account, maintenance equity is $5,000 (25% of $20,000) and the current equity in the account is $8,000 ($20,000 LMV − $12,000 DB). Because the debit balance cannot exceed $15,000, only $3,000 may be withdrawn to keep the equity at the minimum of $5,000.

In a margin account, your customer's long market value is $22,000. The debit balance is $8,000. If the customer enters an order to purchase $12,000 of stock, the margin call will be

$3,000 We determine the excess equity in this account by comparing the equity in the account with the Reg. T requirement. With a LMV of $22,000, Reg. T requires 50% equity, or $11,000. This account has equity of $14,000 ($22,000 minus the $8,000 debit balance). That is how we note the excess equity is $3,000 ($14,000 minus $11,000). Unless stated otherwise, that $3,000 will be journaled to SMA. The margin required on a new $12,000 purchase is $6,000, so this customer would need to deposit an additional $3,000 on top of the SMA.

An ABC 40 call is quoted at 4.25 - 4.50, and an ABC 45 call is quoted at 1.50 - 2.00. What is the cost of establishing a debit spread?

$300 To establish a debit spread, an investor buys a 40 call at the ask price of 4.50 and sells a 45 call at the bid price of 1.50. The net premium paid is (4.50 minus 1.50) times 100 shares, which equals $300.

The XYZ Corporation's A-rated convertible debenture is currently selling for 90. If the bond's conversion price is $40, what is the parity price of the stock?

$36 If the bond's conversion price is $40, it means the bond is convertible into 25 shares ($1,000 par value divided by the $40 conversion price). Parity means equal, so what does each share have to be worth so that 25 of them are equal to $900? Dividing $900 by 25 shares results in a parity price of $36. That does not mean the stock is selling for $36 per share (probably a bit less), but at $36, holding the bond or converting into the stock gives the investor equal value. Some students quickly see that the bond is 10% below its par value, so the stock, to be equal, must be 10% below the conversion price. Take 10% off $40 and the result is $36. Either way works.

A customer is short a DMF 50 call for which he received a premium of 4. Seven months later, the call was exercised when the current market for DMF was 56. Under the Internal Revenue Code, what were the proceeds of his sale?

$5,400 He wrote a call and received a premium of 4. He later sold the security at $50, which made his total receipts for the stock $54. Proceeds in this case refers to the total amount he took in (a $400 premium plus $5,000 upon the sale).

XYZ Technology Fund permits rights of accumulation. A shareholder has invested $9,000 and signed a letter of intent for a $15,000 investment. If his reinvested dividends during the 13 months total $720, how much money must he contribute to fulfill the letter of intent?

$6,000 The shareholder must contribute the full $15,000, so he owes an additional $6,000. Reinvested dividends and changes in the net asset value do not count toward a breakpoint during the period of a letter of intent.

On September 1, an investor sold 100 shares of KLP Corporation common stock for a loss of $1 per share. On September 15, he purchased a KLP convertible bond with a conversion price of $40. How much of the original loss may he now declare for tax purposes?

$75 Because he purchased the convertible bond less than 30 days after realizing the loss, the sale of the stock falls under the wash sale rule: investors who sell securities at a loss and repurchase them, including their equivalents, 30 days before or after the sale will have the loss disallowed by the IRS. With a conversion price of $40, the bond could be converted into 25 shares (1,000 ÷ 40) of KLP common stock. Hence, the investor has bought back the equivalent of 25 shares and may only declare a $75 loss, as the remaining $25 loss will be disallowed.

An investor has an established margin account with a short market value (SMV) of $4,000 and a credit balance of $6,750, with Regulation T at 50%. How much excess equity does the investor have in the account?

$750 The Regulation T requirement and equity must be calculated before excess equity can be determined. The Regulation T requirement is 50% of the SMV of $4,000 ($2,000). Equity is calculated by subtracting the SMV of $4,000 from the credit balance of $6,750 ($2,750). Excess equity is calculated by subtracting the Regulation T requirement of $2,000 from the equity of $2,750 ($750).

One of your customers purchased a fixed premium variable life insurance policy five years ago. The face value of the policy is $2 million and the current cash value is $107,237. The customer calls you and asks about taking a policy loan. Although the exact details are in the prospectus, you know that the minimum amount that could be borrowed is

$80,427.75. Once a variable life insurance policy has been in effect for at least three years, the policy must allow for a policy loan equal to a minimum of 75% of the current cash value.

If an investor sold two BCD Feb 40 calls at 4 on August 4, 2018, and the call expired unexercised, what were the tax consequences?

$800 short-term capital gain for tax year 2019 For tax purposes, any premiums earned are recognized at the expiration date. In this case, the February call options sold in August 2018 for $400 each and expired in February 2019. Uncovered options writers always have short-term gains or losses.

New offering: 800,000 units at $6 per unit. Each unit has two shares of common stock and one warrant. Each warrant is to purchase half a share of common stock. Based on this information, how many shares of stock will be sold, and how many warrants will be sold?

1.6 million shares and 800,000 warrants Warrants may be distributed to stockholders in an underwriting as part of a unit. The warrant is a form of bonus to entice investors to purchase the unit. As each unit contains two shares, 1.6 million shares are being distributed. As each unit also includes one warrant, 800,000 warrants are being distributed.

An investor wishes to invest $5,000 into the KAPCO Balanced Fund, an open-end investment company. How many shares will the investor receive if the next computed NAV per share after receipt of the order is $41.30 and the fund has a sales charge of 4%?

116.225 The investor will pay the POP (public offering price) of $43.02 per share. That price is computed by dividing the NAV of $41.30 by (100% ‒ 4%). Remember, the 4% sales charge is a percentage of the offering price, not the NAV. Dividing the $5,000 investment by the POP of $43.02 results in a purchase of 116.225 shares.

When an account is owned by an individual who is 65 years old or older, or the client is 18 years old or older and a member firm believes the client has an impairment that prevents the person from defending their interests, a temporary hold is permitted on disbursements for how many days if the firm comes to reasonably believe that an attempt at exploiting the person has been made?

15 business days

A registered representative of a FINRA member firm has been found guilty of a trade practice violation. If desired, the Code of Procedure requires the individual to file an appeal within

25 days after receiving the decision. Under the Code of Procedure, appeals must be filed with the NAC within 25 days after receiving the decision.

The minimum maintenance requirement on long stock accounts is what percentage of the market value?

25% The minimum maintenance in a long account is 25% of the market value. It is 30% of the short market value in a short margin account and 50% is the initial margin requirement.

A customer establishes the following positions: Buy 100 ABC at 28 Buy 1 ABC Dec 25 put at 2 What is the breakeven point?

30 The breakeven point is where an investor neither makes nor loses money. In this hedged position, the buyer must recover the cost of the stock and the premium paid to break even (28 + 2 = 30). Please note that the call up and put down rule does not apply when there is a stock position.

Under the USA PATRIOT Act of 2001, member firms must maintain records of reports of currency transactions involving more than $10,000 for

5 years The USA PATRIOT Act of 2001 requires that all currency transactions involving more than $10,000 be reported on a Form 112 Currency Transaction Report and that these forms be maintained for 5 years.

Your customer owns a variable annuity contract. The assumed interest rate (AIR) stated in the contract is 5%. In January, the realized rate of return in the separate account was 7%, and she received a check in February based on this return for $200. In February, the rate of return was 10%, and she received a check in March for $210. For her April check to be $210, what rate of return would the separate account have to earn in March?

5% Each month's payout depends on the actual earnings compared to the AIR. If the actual rate of return equals the assumed interest rate, the check will stay the same. We don't compare one month's return to another's; we compare the actual to the assumed. If the actual is higher, the following month's check goes up. If the actual is lower, the following month's check goes down. And, as stated earlier, if the actual equals the assumed, there is no change.

ZOO is trading at 50.63. Your customer, who owns 100 shares of the stock, places an order to sell ZOO at 50.25 stop limit. The tape subsequently reports the following trades: ZOO 50.63 50.75 50.13 50.17 50.27 Your customer's order could first be executed at

50.27. The sell stop limit order is elected (triggered) at the first trade of 50.13; when the stock trades at or below the stop price of 50.25, the order becomes a sell limit order at 50.25. The order can be executed at that price or higher (the limit placed by the customer). The next trade reported after the trigger is reached is below the limit price. The order could be executed at the next trade of 50.25 or higher, and that is the trade at 50.27.

Trade confirmations must show yield to call on which of the following bonds?

5½%, 5% basis, maturing 2038 Bond confirmations must disclose the lower of the yield to maturity (YTM) or yield to call (YTC). On a premium bond, the YTC is the lower of the two. The terminology here shows the coupon, the basis (YTM), and the maturity date (and, in one case, the call date). The 5½% bond with a 5% basis is the only bond trading at a premium because the YTM (or basis) is lower than the coupon. Even though the 6½% bond maturing in 2059 is callable relatively soon, because the bond was purchased at par, CY, YTM, and YTC are all equal to the coupon (nominal) rate, so the investor won't suffer a loss of principal with an early call.

An investor owns six RIF Apr 150 puts. How many shares of the RIF will change hands if all the options are exercised?

600 Each of the six contracts allows the owner to sell (put) 100 shares of the RIF stock at $150 per share. If all six contracts are exercised, that will be 6 × 100 = 600 shares.

On November 4, a customer writes an S&P 100 Jan 785 put at 6. The maximum potential gain on this position is

600. The potential gain on a short option is the premium received on the transaction.

A bond with a 9% coupon, maturing in 18 years and 6 months, is selling at 120. The yield to maturity is closest to

7.05%. Don't waste time trying to do the yield to maturity computation. This bond is selling at a premium (120% of par). Therefore, all of the computed returns must be lower than the 9% nominal (coupon) yield. Only two of them are. The 7.50% represents the current yield ($90 ÷ $1,200). We know from our charts that, just like a seesaw, the farther from the center you go, the bigger the move at the end. That means the nominal yield is the highest, followed by the current yield (CY), the yield to maturity (YTM), and finally the yield to call (YTC) as the lowest. Because only one choice is lower than the CY, you get the correct answer with minimal effort.

A registered representative calls a dealer for a quote and is given the following quote: Maturity5/28/2019 Bid 2.273 Asked 2.263 Chg.Asked 0.015 yield2.301 What type of security did the registered representative ask for?

A Treasury bill This represents a quote for a T-bill. The quote represents a percentage discount from 100%. It is easy to identify because it looks like the bid price is greater than the ask price. This is the only quote you will see on the Series 7 that looks like this. If you subtract 0.02273 % from 100%, the result is 99.97727% of par, and if you subtract 0.02263% from 100%, the result is 99.97737%. Therefore, the quote is bid $999.7727, ask $999.7737. Note that the ask price is slightly higher. For the exam, it is only important to identify the quote as a T-bill quote; you do not need to do the calculation.

Which of the following statements regarding yield shown on a bond confirmation for a bond that has been called is true?

A bond confirmation will show YTC if the bond has been called under an in-whole call provision. A bond confirmation for a bond called under an in-whole call provision will show yield to call (YTC), as the bond being called away is certain. However, in the event of an in-part call, there is uncertainty as to whether that particular bond will be called. Therefore, the lower of the YTC or yield to maturity would be shown on the confirmation.

There are a number of different types of orders that a registered representative can enter for a client. Of the following, which one would be most appropriate for a client wishing to protect a profit on a short stock position?

A buy stop order Protecting a profit on a short position means covering (buying back) the stock before its price increases above the original sale price. That means buying the stock. The investor would enter the buy stop order with a stop price above the current market, but below the original sale price. A buy limit order is placed below the market, and that is of no help if the price increases. A market order is executed at once, and selling the stock is not appropriate when the investor has already sold it.

If a customer with an unrealized gain on a short stock position wishes to protect her profit, she should enter

A buy stop order can be placed above the current market to protect the short stock position. If the stock trades at or above the stop price, the order is elected and becomes a market order to buy the stock, which will be used to cover the short position.

A convertible corporate bond with a conversion price of $20 is trading at 115. The parity price of the common stock is

A conversion price of $20 means the conversion ratio is 50 ; 1000/20(i.e., each bond can be converted into 50 shares of common stock). $1,150 / 50 = $23 parity price

Which of the following would be defined as a research report?

A document that states the banking industry is ready for recovery but ABC Bank will not participate in the recovery and if owned, investors should sell the security One of the keys to defining a research report is that it suggests taking action (buy, sell, or hold) on the subject security. The term does not include commentaries on economic, political, or market conditions.

Owners of a corporation's common stock who are unable to attend the corporation's annual meeting are

A hybrid REIT receives its name from the fact that it combines the strategy of an equity REIT and a mortgage REIT

Which of the following forms of written communication must a principal approve before use?

A letter sent this month to 50 prospective customers offering advice about a stock Letters sent to more than 25 retail investors within any 30-calendar-day period are considered retail communication and must be approved by a principal before use. Letters sent to fewer than 26 retail investors within a 30-calendar-day period are considered correspondence, which does not need prior approval but is subject to subsequent review.

Which of the following actions would increase SMA in a long margin account?

A long sale Of these choices, an increase in SMA can only be accomplished by a sale of securities held long in the account. A purchase would decrease the SMA if it is used to make the purchase. Stock dividends have no effect on the balances in a long margin account; only the number of shares is changed. A decline in the CMV would not change the SMA.

Which of the following statements best describes a hedge fund?

A private and unregistered investment pool that accepts the investor's money and employs sophisticated hedging and arbitrage techniques using long and short positions, leverage and derivatives, and investments in many markets As of the date of this course, hedge funds are not registered with the SEC (their managers generally are) and they are invariably sold in private offerings, usually under Regulation D of the Securities Act of 1933. Hedging and arbitrage techniques using long and short positions, leverage and derivatives, and investments in many markets are some of the primary techniques used by these funds. Although some hedge funds do outperform the market, a blanket statement cannot be made.

Which of the following accounts would a CMO Z-tranche be best suited for?

A professionally managed hedge fund specializing in real estate portfolio securities A zero tranche (Z-tranche) CMO is considered to be among the most volatile CMO tranches because no payments are received until all preceding tranches of the CMO are retired. Generally, CMO tranches are not suitable for smaller or unsophisticated investors, which is why customers are required to sign a suitability statement before purchasing any CMO tranche. Of the answer choices given, the best suited account would be the one that is professionally managed and already specializing in real estate investments.

Which of the following bodies may not incur overlapping debt?

A state government Overlapping debt is debt of another issuing body that is paid by property taxes of residents. School districts, parks and recreation departments, highway departments, and library systems are all paid through real estate taxes (GOs). State debt is least likely to be overlapping, as states do not generally tax real estate.

An investor wants to invest $20,000 but anticipates needing those funds in five years for a business investment. Currently, with inflation rising, the government is expected to take action to push interest rates up to reduce the money supply. Given these conditions, which of the following securities would be the least suitable for this investor who needs a specific amount of money in five years?

A zero-tranche CMO is subject to interest rate risk as well as extension risk when interest rates rise, and therefore, it would not be suitable for a customer that needs her investment back at a specific point in the future. By contrast, a four-year zero coupon bond will mature within the anticipated time frame for needing the funds and would be the most suitable choice of the answers given.

When market interest rates are rising, the market price of which of the following securities would probably be affected the most?

ABC Gas and Electric, a regulated public utility Stocks that are interest rate sensitive will reflect the impact of a change to market interest rates more than others. Utility stocks, with their high degree of debt leverage and liberal dividend payout ratios, are considered interest rate sensitive. Preferred stock is also subject to interest rate risk, but the adjustable-rate feature minimizes the risk because the dividend rate changes to mirror the market rate. The yield of the money market fund will change, but the combination of the short maturities and the attempt to keep the NAV fixed at $1 per share limits the price risk.

Which of the following would be the least considered factor in determining if a particular type of options trading is suitable for a customer?

Ability to meet margin calls Remember that while most firms require that options trades be done in margin accounts, options purchases are not marginable. Therefore, of the choices listed, the ability to meet margin calls would be the least considered factor regarding suitability.

Advertising relating to municipal securities must be approved by which of the following?

According to MSRB rules, advertising (communications with the public) must be approved by either a municipal securities principal or a general securities principal.

Your new customer lists tax-free income as an investment objective but notes that he will need access to $50,000 within the next four to six months for a down payment on a vacation home he is purchasing. To meet the objective of tax-free income, a registered representative considers municipal securities for the $50,000. Which of the following municipal securities recommendations would be the least suitable?

An auction rate security (ARS) An ARS is a long-term instrument tied to short-term interest rates and, therefore, would not be suitable for someone with a short-term time horizon. Each of the remaining answer choices are short-term notes aligning better with the customer's need to access the funds in the next four to six months.

Which of the following is true regarding an institutional communication?

An individual with $50 million or more in total assets is considered an institution. As long as assets are at least $50 million, it is an institution. Institutional communication is not required to have the preapproval of a principal, nor is it required to be filed with FINRA.

There are certain securities offerings that are limited to those who meet the definition of accredited investor. The SEC requires that the issuer shall take reasonable steps to verify that purchasers of securities sold in those offerings are accredited investors. One way in which this may be accomplished for natural persons is obtaining a written confirmation from certain persons or entities that such person or entity has taken reasonable steps to verify that the purchaser is an accredited investor within the prior three months and has determined that such purchaser is an accredited investor. Confirmation from which of the following would not meet the SEC's requirements?

An investment adviser registered and in good standing under the laws of the state of its principal office It is only investment advisers registered with the SEC, not the state(s), for whom the written confirmation of their accredited investor status is acceptable.

All sell orders must indicate whether they are long or short. In which of the following cases would the sell order be marked long?

An investor has purchased the stock being sold but the trade has not yet settled. A customer is considered long a security only if she owns the stock or has entered into an unconditional contract to buy the stock and will deliver the shares; owns convertible securities, has converted them, and will deliver; or owns an option, has exercised it, and will deliver. Otherwise, the sale is marked as a short sale on the order memorandum.

A customer buys AC Growth Fund and enjoys a substantial paper capital gain. When he believes the market has reached its peak, he switches into AC Income Fund within the AC family of funds. He incurs a small service fee but is not charged an additional sales charge. What is the tax effect?

Any gain in AC Growth Fund is taxable because the exchange is treated as a sale and a purchase. The exchange is treated as a sale of the growth fund shares followed by a purchase of the income fund shares. The gain or loss is determined by comparing the cost basis of the growth fund shares with the net asset value at the time of exchange. Any difference is a capital gain or loss, even though the proceeds were immediately used to purchase the income fund.

Under the Code of Arbitration Procedure, arbitrators fall into one of two categories: public or nonpublic arbitrators. Which of the following persons could not be a public arbitrator?

Any person who worked in the financial industry for any portion of her career Those who have worked in the financial industry for any duration during their careers will always be classified as nonpublic arbitrators. Certain professionals, such as attorneys and accountants, can be considered nonpublic if at least 20% of their revenues comes from representing financial industry firms or any parties to an arbitration. One time in five years is certainly not going to be 20% of their revenues.

Customer account statements must include the per-share estimated value of a direct participation program (DPP) or unlisted real estate investment trust (REIT) security held in the account. Which of the following does FINRA accept as an estimated value methodology?

Appraised value For these securities, where a ready market does not exist, the estimated value must be based on an appraised valuation of the assets and liabilities of the DPP or REIT. This appraisal must be performed at least annually, by, or with the material assistance or confirmation of, a third-party valuation expert or service. The value must be derived from a methodology that conforms to standard industry practice. Because there is no market value, an alternative, such as appraisal, must be used. There is another method known as net investment, not gross investment. Tax basis is irrelevant to the current value.

Your customer, a small-business owner, likes investments that are short term, relatively safe from credit risk, and liquid. He's heard that higher rates of return can be realized from auction rate securities than the rates he is currently getting on the Treasury bills in his portfolio. He asks you to explain them to him. Which of the following would you note as being reasons why they are not suitable for him?

Auction rate securities are intended as long-term investments. If the auction fails, holders of ARSs may not have immediate access to their funds. Auction rate securities (ARSs) are long-term variable rate bonds with maturities of 20 to 30 years tied to short-term interest rates. As long-term instruments, they are not suitable for an investor favoring short-term investments. Additionally, interest rates are reset using a Dutch auction method at predetermined intervals, typically 7, 28, or 35 days. A failed auction can occur due to lack of demand; in this case, no bids are received to reset the rate. This risk would not align the investment objectives of safety and liquidity.

A corporation is having a rights offering. The terms of the offering require six rights plus $60 to purchase one share. With the stock's current market price at $74 per share, the theoretical value of one right before the ex-rights date is

Because the question is asking about the value before ex-rights, it means we use the cum-rights (with rights) formula; that is, the market price minus the subscription price divided by the number of rights it takes to buy one share plus one. Plugging in the numbers gives us ($74 - $60) ÷ (6 + 1) = $14 ÷ 7 = $2.00.

Which of the following oral orders can be accepted from a customer without additional documentation?

Buy 100 shares of ABC when the price is right Prices and time of execution do not require discretionary authority.

A customer believes ABC's stock price will rise, but she does not currently have the money to buy 100 shares. How could the customer use options to profit from a rise in the stock's price?

Buy calls and write puts When the price of a stock that underlies a call option increases in price, the owner (holder) of that option stands to profit. An investor who has sold a put option on that stock will also benefit because the option will expire unexercised and the writer will get to keep the premium.

An investor purchases 100 shares of JKL common stock at a price of $42 per share on April 22, 2018. On June 27, 2019, JKL's market price is $51 and the investor liquidates the position. Which of the following transactions made on October 17, 2018, would have an effect on the investor's tax treatment of this gain?

Buying a Feb 45 JKL put Long-term capital gains tax rates are available when one has a holding period of more than 12 months. Although this investor held the JKL stock for more than 14 months, the purchase of the put caused the holding period to be tolled (the IRS term for suspended). It means that the holding period from April 22 to October 17 (almost 6 months) is put on hold until the put is disposed of or expires. When that happens in February, the "clock" picks up where it left off and runs another 4+ months until June 27. The total time period is approximately 10 months, less than the 12 months required for long-term treatment. None of the other positions affects the holding period of a long stock position.

If an investor purchases 500 shares of an aggressive growth stock, which strategy would limit his downside risk?

Buying five puts on the stock A put gives the investor the right to sell stock at a set price (the strike price) for a period of time, and it protects against losses below the strike price. Buying calls can protect a short stock position. If the customer is long stock, the purchase of calls on that security increases leverage and risk. Writing a put creates the obligation to buy more stock at the strike price, which increases downside risk.

Under SEC rules, which of the following events requires a broker-dealer to furnish an updated account statement to a customer?

Change of the customer's name or address Any change in a customer's status, particularly one that could be a sign of potential identity theft, requires a broker-dealer to update the customer account record. A copy of the change must be furnished to the customer within 30 days for verification purposes.

An investor has unexpectedly received $30,000 from an old debt he had written off. This money will come in handy for a business venture planned for three years from now. Meanwhile, he would like to generate some income on the money with as little risk and as little expense as possible. Which of the following recommendations is likely to be the most suitable for this customer?

Class C shares of the ABC Investment-Grade Bond Fund The customer wants income with as little risk as possible, so our answer must be one of the choices that offer an investment-grade bond fund. Of those offered, Class C shares would be best, because the customer would pay no front-end sales charge and no CDSC after a short time, probably one year. He will pay somewhat higher 12b-1 fees than with Class A shares, but this will amount to only a fraction of 1% per year, and only for the three years of his investment.

A registered investment company whose share price fluctuates independently of its net asset value is most likely

Closed-end funds' share prices can differ significantly from their NAVs. Open-end (mutual) fund shares are purchased and redeemed at their NAVs. UITs are redeemable at NAV

Which of the following must be registered as investment companies under the Investment Company Act of 1940

Closed-end investment companies Separate accounts of insurance companies offering variable products Under the Investment Company Act of 1940, face amount certificate companies, unit investment trusts, open- and closed-end management companies, and separate accounts of insurance companies used to fund variable annuity and variable life contracts must register with the SEC as investment companies. Note that the separate account is registered as an investment company, not the variable contract.

A customer is very concerned about investments that may not keep pace with inflation. He asks which securities would have the least exposure to inflation risk. Which of the following would be the best answer?

Common stock The returns on common stock have historically outperformed inflation, making them less vulnerable to loss of purchasing power than the other choices presented. Cash is a store of present purchasing power that inflation will erode. Fixed annuities have more exposure to inflation than common stock because their payments are fixed in nominal dollars. Preferred stock has the same exposure to inflation risk as do all fixed-income instruments.

When it comes to issuing a debt security, which of the following features will generally enable the issuing corporation to borrow at the lowest interest rate?

Convertible Because the convertible feature offers potential growth through the exercise of the conversion option, the interest rate on these securities is generally lower than other debt issues of the same corporation. The call feature increases the reinvestment risk and that is compensated for with a higher coupon. The descriptive adjective cumulative refers to dividend payments on preferred stock, but not to bonds. Because zero-coupon bonds pay nothing until maturity, that added risk requires a higher yield to attract investors.

An investor is looking for a fixed-income investment that can provide a reasonable income while offering potential inflation protection. Which of the following would be the best recommendation to meet this investor's objective?

Convertible bonds offer the best solution for this client. The bond carries a fixed interest rate, meeting the goal of reasonable income. The ability to convert the bond into common stock offers the potential to keep pace with inflation.

Section 28(e) of the Securities Exchange Act provides a safe harbor for certain soft dollar compensation extended from broker-dealers to investment advisers. Which of the following is most likely to be included in that safe harbor?

Customized software designed to give clients access to asset allocation programs Among the items generally in the safe harbor are those items designed to assist the firm's customers. Customized software that helps clients would be acceptable. Although seminar registration expenses are in the safe harbor, travel and transportation expenses, such as meals and lodging, are not. Rent and office furniture are specifically listed as out of the safe harbor.

Government agency bonds issued by which of the following carry a minimum denomination of $1,000 with $1 increments?

Ginnie Mae GNMA securities are available with a minimum denomination of $1,000 and in increments of $1.00. That means, for example, that a client can purchase one for $1,003.00 or $1,337.00 if desired. The only other agency with that type of pricing is the FNMA.

A quote of 2.20 bid 2.18 ask would most likely be a quote on

Discounted instruments (such as T-bills) are quoted on a discount-to-yield basis. Even though the number representing the bid is higher than the ask, it would be lower when converted into dollars. That is, the asking price is the face amount less a discount of 2.18%, while the bid price is the face amount less a discount of 2.20%. Just like anything we buy, the greater the discount, the lower the price.

When it comes to securities data, the industry relies heavily on acronyms. Which of the following is used exclusively for municipal securities?

EMMA is the Electronic Municipal Market Access system. EMMA is a centralized online source for free access to municipal disclosures, market transparency data, and educational materials about the municipal securities market and is operated by the MSRB. ACT is the Automated Confirmation Transaction Service. TRACE is the Trade Reporting and Compliance Engine. OATS is the Order Audit Trail System. All of these serve various parts of the industry, but EMMA is the only one exclusively devoted to municipal securities.

Asset allocation refers to the spreading of portfolio funds among different asset classes with different risk and return characteristics. When allocating among asset classes, you would not include

ETFs are a way of investing in equity (stock) or debt (bonds) securities and are not a separate asset class.

ABC Company has issued $20,000,000 of convertible bonds with a coupon of 5% and a current market value of 120. The conversion price is $40. If all the bonds are converted, how many additional shares of common stock will ABC have outstanding?

Each bond will convert to 25 shares of common stock ($1,000 ÷ $40). 20,000 bonds were issued ($20,000,000 ÷ $1,000). Therefore, 500,000 additional shares (20,000 × 25) will be outstanding if all the bonds are converted.

Which of the following responsibilities did the Municipal Securities Rulemaking Board (MSRB) receive through the Securities Acts Amendments of 1975? Regulation of municipal issuers Establishment of recordkeeping requirements for municipal broker-dealers Enforcement of any municipal regulations it adopts Creation of regulations for participants in the municipal securities secondary market

Establishment of recordkeeping requirements for municipal broker-dealers Creation of regulations for participants in the municipal securities secondary market

FINRA Rule 2310 defines a direct participation program as "a program which provides for flow-through tax consequences regardless of the structure of the legal entity or vehicle for distribution including, but not limited to, oil and gas programs, real estate programs, agricultural programs, cattle programs, condominium securities, Subchapter S corporate offerings and all other programs of a similar nature, regardless of the industry represented by the program, or any combination thereof." The rule places limits on the amount of broker-dealer sales compensation considered fair and reasonable. That limit is

FINRA limits the amount of the sales compensation to 10% of the gross proceeds of the offering.

All of the following statements regarding the Federal National Mortgage Association (FNMA) are true except

FNMA is owned by the U.S. government. FNMA is a publicly held corporation; it is not owned by the federal government. The interest income on all mortgage-backed securities is fully taxable on the local, state, and federal levels. Though FNMA is a government agency, FNMA pass-through certificates are not guaranteed by the U.S. government. The only tested U.S. agency whose securities are considered direct obligations of the U.S. government is the Government National Mortgage Association (GNMA).

If an investor is in the highest federal income tax bracket and is subject to the alternative minimum tax (AMT), which of the following securities should an agent recommend?

General obligation (GO) bond Municipal bonds are suitable for the portfolio of an investor who is in a high tax bracket because the interest is exempt from federal income tax. A GO bond is a better recommendation than an industrial revenue bond because the interest on industrial revenue bonds is likely subject to the AMT.

Dale Wells, a British citizen temporarily working in the United States, wants to form a business venture with other investors. Wells is looking for favorable tax treatment of earnings and losses. Wells also wants to limit the number of investors but is willing to share control of the enterprise with others to attract them. What business form would you advise?

General partnership Limited partnerships would not work because the other investors have limited say in how the enterprise is run. C corporations do not provide favorable tax treatment of gains or losses. Although an S corporation appears to be the right answer, only U.S. citizens or resident aliens can own one.

If an investment representative hosts an investment seminar and intends to discuss general investment concepts and a specific mutual fund for which he has performance charts, which of the following are true?

He may discuss the investment returns of the mutual fund using a specific time frame. He must disclose all material facts regarding the mutual fund to the audience.

Which of the following statements regarding hedge funds is correct?

Hedge funds are usually structured as a partnership. Hedge funds are usually structured as a partnership, with the general partner as the investment manager and the investors as limited partners. Hedge funds are actively and aggressively managed, seeking superior returns, and they are best suited for wealthy, sophisticated investors. Under the typical 2% + 20% fee schedule, hedge fund managers are largely compensated for performance, not assets under management.

A customer who seeks to supplement his retirement income and has a high risk tolerance would find which of the following securities most suitable?

High-yield bond funds High-yield bonds yield more than investment-grade bonds. Because the client has a high risk tolerance, these bonds are more appropriate than investment-grade bonds, which yield less.

Which of the following statements regarding ADRs is not true?

Holders have voting rights (not for ADRs)!!!

The GHI Transportation Company has run into decreased sales and is forced into a bankruptcy liquidation. Which of the following would have the most junior claim?

Holders of GHI commercial paper Secured debt (such as the mortgage bond), the collateral trust certificate, and the equipment trust certificate have first priority in the event of a bankruptcy. Commercial paper is a promissory note relying on the creditworthiness of the issuer.

In a strong bull market, which of the following positions utilizing leverage has the potential for the highest percentage gain?

Holding calls Both a long call and a long stock position are profitable in a rising market. However, because options use leverage, the profit relative to the money invested is larger with option positions. A put writer also profits in a rising market, but only by the amount of the premium. A short seller loses money if the stock rises.

A husband and wife wish to open a Roth IRA. She is 49 years old and earns $99,000 per year; he is 51 and earns $49,000 per year. What is the maximum permitted contribution for the married couple, based on age and income?

Husband $7,000 and wife $6,000 The husband may contribute $7,000 and his wife may contribute $6,000. For married couples, an adjusted gross income level of $196,000 (2020) begins to limit the amount of contribution that is permitted into a Roth IRA (although this specific amount is never tested). The married couple has an income level of $148,000, well below that AGI. Therefore, each would be permitted to make the maximum contribution. The wife may contribute a maximum amount of $6,000. Because the husband is 51, he is eligible to contribute an additional $1,000 per year (the catch-up provision applied to those age 50 and older) for a contribution of $7,000.

A municipal bond in default is in good delivery form if I past-due and current coupons are attached. II the bond is insured. III subsequently due coupons are attached. IV the issuer files a default guarantee letter with the Municipal Securities Rulemaking Board.

I and III To be in good delivery form, a municipal bond must be accompanied by all unpaid coupons: past due, currently due, and subsequently due. Insurance or letters of guarantee do not constitute good delivery.

Which of the following statements are true? Build America Bonds (BABs) are tax exempt at all levels. Direct-payment BABs provide the municipal issuer with payments from the U.S. Treasury. BABs are issued by the U.S. Treasury. Tax credit or issuer BABs provide the municipal bondholder with a federal income tax credit.

II and IV Created under the Economic Recovery and Reinvestment Act of 2009 to assist in reducing costs to issuing municipalities and to stimulate the economy, BABs are taxable municipal securities. There are two types: direct payment BABs that provide the municipal issuer with payments from the U.S. Treasury and tax credit or issuer BABs that provide the bondholder with a federal income tax credit.

In an account opened by two individuals as joint tenants with right of survivorship, all of the following are true except

In a JTWROS account, each party has an equal, undivided interest in the account. Upon the death of one party in a two-party account, the other party assumes full ownership of the account. Orders may be entered by either party, and mail may be directed to either party. However, disbursements of cash or securities must be in the name of all parties to the account.

A brother and sister would like to open an account together. Contributions to the account will be disproportionate. Both the brother and sister have children and they each want their children to get their proportionate shares when either the brother or sister dies. What type of account would you recommend?

In a tenants in common account, a deceased tenant's fractional interest in the account is retained by that tenant's estate and can be passed to the deceased's heirs and not to the surviving tenant(s).

One of your clients has $30,000 in the Balfour Balanced Fund in her personal account. She is preparing to invest an additional $12,000. Balfour has a breakpoint at $50,000. The client mentions that she has other holdings in Balfour and wonders if they will count toward the breakpoint. You respond that all of the following accounts would qualify except

In most cases (all on the exam), a mutual fund will allow investors to get a breakpoint discount by combining their fund purchases with those of their spouse and minor children. They are also able to credit mutual fund transactions in retirement accounts, educational savings accounts, or even in accounts at other brokerage firms.

If general interest rates increase, the interest income of an open-end bond fund will do which of the following?

Increase What does a bond fund invest in? Bonds. One of the features of an open-end company (mutual fund) is the continuous issuance of new shares. The question states that interest rates are increasing. As that new money is received, the fund's manager will be able to invest in bonds offering that higher return. This will cause the income of the fund to increase. It is not part of the question, but the increase in interest rates will likely lead to the NAV of the fund decreasing (as interest rates go up, the price of the bonds in the fund's portfolio will go down).

In analyzing a municipal government obligation bond, an increase in all of the following would be a negative indication except

Increasing property values would have a tendency to increase the taxes paid to the municipality.

Which of the following statements regarding revenue bonds issued by a state or municipality is true?

Interest will be paid only if the enterprise owned and operated by the state or municipality has sufficient earnings to cover the interest payments or the debt service reserve. Because revenue bonds are not backed by the full faith and credit of the municipality that issues them, the earnings of the revenue-producing project must be large enough to cover the interest and principal payments.

If TCB is trading at 43 and the TCB Apr 40 call is trading at 4, what are the intrinsic value and the time value of the call premium?

Intrinsic value: 3; time value: 1 The option is in-the-money by 3 points because the strike price is 40 and the market price is 43. This sets a minimum premium of $3 per share. Because the actual premium is 4, the balance of 1 represents time value. The premium, minus the intrinsic value, equals the time value. This is true whether the option is a put or a call.

A resident of New York City purchases an Albany, New York, general obligation bond and receives $600 of interest from that bond during the year. How is that $600 taxed?

It is not subject to federal income tax. Interest from public purpose municipal bonds is exempt from federal income tax, and most states have chosen to make interest on their municipal bonds exempt from state income tax to residents of their states.

If a client asked you about an equity security with a cumulative feature, the question would most likely be dealing with

It is preferred stock that can offer the cumulative feature

Which of the following is a bull spread?

Long Jul 30 put, short Jul 35 put A debit call spread is bullish and a credit put spread is bullish. Long Jul 30 put, short Jul 35 put is the only bullish position in the answer choices. Short Aug 40 call, short Aug 40 put is a short straddle, not a spread, and the remaining two positions are bearish: long Aug 30 call, short Aug 25 call and long May 40 put, short May 35 put. Remember, buying the low strike and selling the high strike is always a bull spread, regardless of the spread being puts or calls.

In the trading of options, there are a number of different multiple option strategies. An investor has the following position: Buy one RIF Apr 120 call Buy one RIF Jul 130 put Which strategy is the investor using?

Long combination A combination is composed of a long call and long put, or a short call and a short put, each having different strike prices and/or expiration months on the same underlying security. A straddle is when the expiration dates and exercise prices are the same. A spread consists of a long and short position in the same options class (two puts or two calls). In a diagonal spread, the exercise price and the expiration dates are different. In a time spread, everything is the same except the expiration dates.

Your client's position is long 100 MNO purchased at 90. Which of the following strategies will limit the customer's loss to $700?

Long one MNO 90 call at 4, long one MNO 90 put at 3 It is the long put in this straddle position that limits the maximum loss on the long stock position. If the MNO stock drops to $0, the customer loses $9,000 on the long stock position but retains the right to sell the stock to someone at $9,000, to prevent loss beyond the premium of $300. The call would expire out of the money, for a total loss of $700 (from two premiums)

Which of the following is always affected by a change in the market value of securities in a long margin account?

Maintenance requirement SMA is only affected if the current market value (CMV) increases. In terms of dollars, the maintenance requirement will continuously fluctuate with the market value because it is a percentage of the CMV.

An investor sells an uncovered GHI Aug 120 call for 5½ points. What is the investor's maximum gain, maximum loss, and breakeven point?

Maximum gain = $550; maximum loss is unlimited; breakeven point = $125.50. The initial action is a sale resulting in a credit of $550. That credit is the most the investor can make. Because this is an uncovered call, the maximum loss occurs when the stock rises. Because there is theoretically no upper limit on a stock's price, the maximum potential loss is unlimited. Breakeven follows the call-up rule. Add the $550 premium to the $120 strike to arrive at $125.50.

A registered municipal bond salesperson at your firm has obtained discretionary power for the account of a physician in Gloucester County, New Jersey. The customer is conservative, avoids investment risk, and seeks principal with long-term growth potential. Given the following choices, the salesperson would most appropriately invest the customer's money in

New Jersey Turnpike revenue bonds rated AA. The Michigan revenue bonds, the subinvestment-grade municipal bonds, and the Delaware municipal bonds have possible state disadvantages or are less than investment grade.

A unit investment trust has 90% of its portfolio invested in high-grade bonds with an average maturity of almost 25 years. If the industry consensus was that long-term interest rates were about to increase sharply, which of the following actions would most likely be taken?

No action would be taken One of the key distinctions of a UIT is its lack of management. Once the portfolio has been created, it is fixed until maturity, in the case of debt securities, or until some predetermined liquidation point, in the case of an equity trust.

Toby Jensen originally purchased 400 shares of CSC stock on margin at a price of $60 per share. The initial margin requirement is 50%, and the maintenance margin is 25%. CSC stock price has fallen dramatically in recent months, and it closed today with a sharp decline, bringing the closing price to $40 per share. Based on FINRA requirements, will Jensen receive a maintenance margin call?

No, the account meets the minimum maintenance margin requirement. Minimum maintenance requires equity equal to 25% of the current market value. If the price per share is $40, then the value of the 400 shares is $16,000. That would make the minimum equity requirement $12,000. The initial purchase was $24,000 resulting in a debit balance of 50%, or $12,000. If the LMV is $16,000 and the debit balance is $12,000, the account is right at the minimum maintenance level. Toby might be below the house maintenance, but because that is not given in the question, there is no way to tell.

Investors in zero-coupon corporate bonds would find all of the following to be true except

On a corporate zero-coupon bond, the discount is accreted on an annual basis and investors receive a Form 1099-OID indicating the amount of taxable accretion earned for the year. That is one of the reasons why these bonds are favored for tax-sheltered accounts, such as an IRA, or for UTMA accounts where the child's income might be very low. One of the important characteristics of these bonds is that their duration is equal to the length to maturity. This gives them a longer duration than coupon bonds of the same length and is the reason for their greater price volatility.

A new customer has given you written authorization to transfer the holdings in his account at another broker-dealer to his new account at your broker-dealer. Under the Uniform Practice Code, using the Automated Customer Account Transfer Service (ACATS) Form, the carrying broker-dealer would have how many days to validate the positions?

One business day Under the Uniform Practice Code, the carrying broker-dealer has one business day to validate positions and three business days to transfer to the receiving broker-dealer after validation.

Which of the following investment companies registered under the Investment Company Act of 1940 can include senior securities in its capital structure?

Only the closed-end company is legally permitted to issue senior securities (preferred stock and bonds).

SEC regulations for securities issued by investment companies prohibit which of the following?

Open-end funds from issuing preferred stock Open-end funds from issuing bonds Closed-end funds may issue more than one class of security, including debt issues and preferred stock. Open-end funds may issue only one class of security: redeemable, voting common stock. They may not issue senior securities.

A client has purchased a nonqualified variable annuity from a commercial insurance company. Before the contract is annuitized, your client, age 60, withdraws some funds for personal purposes. What is the taxable consequence of this withdrawal to your client?

Ordinary income taxation on the earnings withdrawn until reaching the owner's cost basis Contributions to a nonqualified annuity are made with the owner's after-tax dollars. Distributions from such an annuity are computed on a last-in, first-out basis, with the income taxed first. Once the cost basis is reached, any further withdrawals are a nontaxable return of principal. Because the client is older than 59½ at the time of distribution, the additional 10% penalty tax is not incurred.

A client interested in the returns offered by CMOs asks you which type has the lowest prepayment risk. What should you say?

PACs Although there can be exceptions, in general, the planned amortization class (PAC) has the lowest prepayment risk. The Z-tranche is the most unpredictable because it is paid off only after all of the other tranches.

Which of the following mortgage-backed securities would provide investors with the most predictable maturity date?

PACs PACs are planned amortization class CMOs and have established maturity dates. Prepayment risk is transferred to the PAC companion, or support, class bonds.

In discussing a direct participation program with your customer, she notes investment characteristics that are important to her and some that are not. For a DPP to be considered suitable for the customer, rank the following items in order of those that should be most important to those that should be least important.

Potential for economic gain, Tax write-offs, Liquidity and marketability, SEC approval. In the eyes of the IRS, a program's economic viability should be the most important aspect of the investment for a limited partner and the first priority in the assessment of the DPP. While the IRS considers programs designed solely to generate tax benefits abusive, they do allow for some in terms of writing off passive income and allowable tax credits, so these factors would be the next concern for an investor. Because there is a very limited secondary market for DPPs, liquidity and marketability should be a low priority, and because there is no SEC approval of any investment, it would be of no concern.

Which of the following is not considered a debt security?

Prior lien preferred stock. Stock, whether preferred or common, represents equity (ownership) and is never considered a debt security.

All of the following statements about the special memorandum account (SMA) are true except

SMA in a long margin account decreases when the market value decreases. The amount of SMA in a long account decreases only when it is used, and is unaffected by market value decreases.

When discussing a client's finances, which of the following would be of least importance when planning to make a lump-sum investment?

Salary enables the registered representative to determine the funds available for periodic investment. A lump-sum investment could be made with money from an inheritance, a year-end bonus, or lottery winnings.

A stock is trading consistently between $20 and $24. The investor with a long position is neutral on the stock. The goal is to generate income. Which of the following recommendations is most appropriate?

Sell a call The investor should sell a call on the stock and collect the premium (income). The investor is long the stock, so it would be better if the price goes up rather than down. Therefore, the sale of a call is better than the sale of a put, and those are the only real choices when the investor wants income through options.

Your customer has contributed $1,000 annually into her Roth IRA for seven years. Which of the following statements concerning her Roth IRA distributions is true?

She will not be taxed on the distributions if she is over age 59½ and the money has been held in the account for five years beginning with the first tax year for which a contribution was made to any Roth IRA established for her.

Which investor has the greatest potential risk if the price of QRS goes up?

Short 10 calls on QRS If the market is rising, the greatest potential risk that an investor can take is a short naked call because the potential loss is unlimited. Please note that any short option, put or call, is uncovered unless something in the question indicates the opposite.

A new client asks for your advice in investing an inheritance of $50,000. The client is 35 years old, has an annual salary of $40,000, and is married with two children. Which of the following would be the least likely suitable recommendation?

Splitting $25,000 into two different AAA-rated GO municipal bonds We do not know a lot about this client, but, with an annual income of $40,000 and two children, the tax bracket is in the lower end. Certainly for exam purposes, municipal bonds are never the suitable recommendation unless something in the question indicates that the client is in a high tax bracket. As far as all $50,000 into one mutual fund, the nature of a balanced fund is that its assets are broadly allocated between equity and debt securities so that it accomplishes the diversification shown in the other choices.

A customer, currently finding the income offered from a money market fund quite low, asks if there might be any debt instruments providing income that one could expect to at least keep pace with inflation as well as offer some tax relief. What suitable recommendation could be made that meets the investor's investment objectives?

TIPS Treasury Inflation Protection Securities (TIPS) are specifically designed to provide income that keeps pace with inflation. In addition, the interest is tax exempt at the state and local level, providing some tax relief

Which of the following securities is frequently offered with a 50-year maturity?

TVA bonds Tennessee Valley Authority bonds are the only government security available today with a maturity as long as 50 years. Most of the agencies don't offer anything longer than 20 years, and the maximum on Treasury bonds is usually 30 years.

Market timing is normally associated with which of the following portfolio management styles?

Tactical asset allocation, which attempts to capitalize on short-term market swings, is a market timing strategy.

A legal opinion issued for a municipal bond covers which of the following?

Tax status of the municipal debt Constitutionality and legality of the municipal debt Municipal securities are reviewed by specialized lawyers who render a legal opinion. The opinion covers two main issues: constitutionality (i.e., it ensures that the bonds are legal, valid, and binding obligations of the issuer) and verification of the tax status of the debt (i.e., interest on the bonds is exempt from federal income taxes as well as state and local taxes in some cases).

Which of the following statements regarding the suitability of municipal bonds are true?

Tax-free interest payments are more suitable for those for whom the tax advantage has the most impact. That would be those in higher tax brackets, who would pay more taxes on the interest received if the interest payments were taxable. Additionally, the tax-free interest is why municipal bonds are not suitable for retirement accounts. This is because the earnings in retirement accounts are already tax deferred, and the impact of receiving tax-free interest is lost or diminished.

A customer owns 10M of 7% U.S. Treasury bonds. He is in the 28% federal tax bracket and the 10% state tax bracket. What is his annual tax liability on these bonds?

The 10M means $10,000. (Remember your Roman numerals? M equals 1,000). His tax liability is as follows: $1,000 times 7% equals $70 annual interest per bond; $70 times 10 equals $700 annual interest, which is taxable only by the federal government; and $700 times 28% equals a $196 tax liability.

With ABC stock selling for $49, a client sells one ABC 50 Nov call option in his cash account with your firm. One week later, ABC is now at $51 per share and his spouse sells two ABC 50 Nov calls in her account. In early November, ABC is selling for $62 per share and the spouse is assigned an exercise notice on one of the calls. The client calls and asks you, "Why was the exercise notice assigned to my spouse and not me?" You should respond:

The OCC uses random allocation when assigning exercise notices. When an option is exercised, the OCC determines to whom it will be assign by random selection. Broker-dealers can elect to use that or FIFO. The size of the position is never taken into consideration, nor is the market price at the time of the write. LIFO is not an acceptable manner for assigning options.

The certificate of limited partnership contains which of the following?

The amount of time the partnership expects to exist All conditions of dissolution The certificate contains, among other information, the limited partnership's name and business, the amount of time the partnership intends to exist, and the conditions of dissolution. It does not contain each partner's net worth, nor is there a market-out clause like those generally associated with underwriting agreements for new issues.

In July, a customer invested $10,000 in the ABC Mutual Fund. In December of the same year, ABC announced a long-term capital gains distribution. In May of the next year, the customer decided to redeem his shares for a capital gain. How are both of the capital gains treated for tax purposes?

The capital gain distribution is treated as long term and The capital gain from redemption is treated as short term. When long-term capital gains are distributed, the length of time an investor has owned the fund is not relevant; it's still a long-term distribution. However, redemption of shares follows the normal holding period rules. Therefore, when this customer sold shares 10 months (July to May) after the purchase, the gain, like any other gain from a holding period that does not exceed 12 months, is short term.

A customer purchases an ABC 6.5% convertible preferred stock at $80. The conversion price is $20. If the common stock is trading two points below parity, the price of ABC common is

The conversion ratio is computed by dividing par value by the conversion price ($100 par / $20 = 5). Parity price of the common stock is computed by dividing the market price of the convertible by the conversion ratio ($80 / 5 = $16). $16 − 2 = $14.

Index options differ from stock options in which of the following ways?

The exercise settlement is in cash. When an index option is exercised, cash is paid to the option holder for the amount in the money. In contrast, exercising a stock option involves delivering the underlying stock. Both index options and stock options have the same expiration date and the same trade settlement date. Closing transactions can be purchases or sales for any option, regardless of the underlying asset.

Which of the following events requires a member firm to provide a client with a specific FINRA-designed educational communication?

The firm, directly or through a registered representative, individually contacts a former customer of that registered representative to transfer assets to the firm. Registered representatives who leave their firms often contact former customers and emphasize the benefits the former customers would experience by transferring their assets to the firm that recruited the registered representative (recruiting firm) and maintaining their relationship with the representative. In this situation, the former customer's confidence in and prior experience with the representative may be one of the customer's most important considerations in determining whether to transfer assets to the recruiting firm. However, former customers may not be aware of other important factors to consider in making a decision whether to transfer assets to the recruiting firm, including direct costs that the customer may incur. Therefore, to provide former customers with a more complete picture of the potential implications of a decision to transfer assets, Rule 2273 requires delivery of a FINRA-created educational communication by the recruiting firm highlighting key considerations for former customers in transferring assets to the recruiting firm, and the direct and indirect impacts of such a transfer on those assets.

An investor with no other positions buys 1 DWQ May 75 call at 6.50. If the investor exercises the call when the stock is trading at 77 and immediately sells the stock in the market, what is the investor's profit or loss?

The investor exercised the right to buy the stock for 75 and can sell the stock in the market for 77, for a gain of 2. The investor paid a premium of 6.50 minus the gain of 2, which gives the investor a loss of 4.50 (4.50 × 100 = $450).

Which of the following statements regarding limited partnerships are true?

The maximum commission in selling partnership offerings is 10%. Commissions taken are not deducted from the original investment to determine beginning cost basis. Under FINRA rules, the maximum compensation that can be taken by sponsors selling direct participation programs is 10%. Up-front costs, such as commissions taken and accounting costs, do not reduce the beginning cost basis.

Which of the following statements describing Section 529 plans is true?

The maximum lifetime contribution varies from state to state. The features of Section 529 plans, including their contribution limits and fees, vary widely from state to state. Section 529 plans have no age limits as to participation; they are open to both children and adults who plan to attend college or graduate school. For college savings plans, there is no state residency requirement for either owners or beneficiaries of Section 529 plans

In April, a customer buys 1 MCS Oct 50 call for 9 and sells 1 MCS Jul 50 call for 4. What will the customer's profit or loss be if he buys back the July call for $1 and sells the October call for $12?

The net gain is $600 because the client paid a total of 10 (9 + 1) and received a total of 16 (12 + 4).

Your clients' option position has been adjusted due to a 2-for-1 stock split. Which of the following regarding this 2-for-1 adjustment is true?

The number of contracts owned will increase. For even splits (i.e., 2 for 1 or 3 for 1), the number of contracts owned will increase proportionately. The number of shares per contract will remain unchanged, and the strike price will decrease proportionately.

When a municipal bond has a net revenue pledge, what is the first item that gets paid from the revenue received?

The operations and maintenance fund. Under a net revenue pledge, operations and maintenance expenses are paid before all debt service. Payments are made in the following order: operating and maintenance expenses, debt service, debt service reserve, and surplus.

Prior to effecting an initial penny stock transaction for a new customer, the registered representative must do all of the following except

The question tells us this is the first trade in penny stocks for a new customer. The monthly statements haven't started yet. The penny stock rules require registered representatives to provide disclosure information to all penny stock buyers. In addition, they must determine suitability based on financial information, investor experience, and objectives supplied by the buyer. As additional protection, the customer must sign the suitability statement.

SEC Rule 17a-4 describes the retention requirements for records made by registered broker-dealers. Which of the following records has a retention requirement of six years?

The record of a transfer of a customer account to another member firm When using the ACATS system to transfer a customer account from one member to another, the transferring member must keep records of the transfer for six years. The reason is that customer account records have a six-year retention requirement, and the transfer documentation contains the account information. Customer complaints are a four-year record—and corporate documents, such as charters or articles of incorporation, are for the lifetime of the firm. Trade confirmations are a three-year record, regardless of the nature of the customer.

Under the Investment Company Act of 1940, which of the following statements regarding the renewal provisions of an investment adviser's contract is not true?

The renewal may be executed orally, provided it is done within two years of the initial contract. When an investment company employs an outside investment advisory firm to manage its portfolio, the act requires a written contract setting forth the adviser's compensation. The contract is for two years initially and must be renewed annually thereafter. The contract must be initially approved by a majority vote of the outstanding shares and the noninterested members of the board of directors and annually renewed by either a majority vote of the board of directors or of the outstanding shares, as well as a majority vote of the noninterested members of the board. The contract must be terminable at any time, with a maximum of 60 days' notice and with no penalty, upon a majority vote of the board of directors or of the outstanding shares, and it must terminate automatically if assigned.

When XYZ stock trades at 40, and an XYZ Oct 35 call trades at 5, which of the following is true?

The time value is zero. An option's premium consists of time value and intrinsic value. In this situation, the call is in the money by 5 (intrinsic value is 5), because the market value of 40 exceeds the strike price of 35 by 5. If the total premium is 5, and the intrinsic value is 5, the time value must be zero. The option is at parity, which means the premium equals the intrinsic value. Remember P - I = T (Premium minus intrinsic value equals the time value).

An investor purchased a corporate bond with a 6% coupon at a net price of 101. The bond had accrued interest for 45 days. Which of the following statements regarding the confirmation of this trade is correct?

The total amount due on the purchaser's confirmation will appear as $1,017.50. Accrued interest is always added to the price of a bond. When you buy the bond, you pay that accrued interest, and when you sell a bond, you receive that accrued interest. The principal value is 101, or $1,010. Forty-five days of accrued interest is ⅛ of a 360-day year, or ¼ of a 180-day semiannual interest payment. With a 6% coupon, the bond pays $30 every 6 months. One quarter of that is $7.50 so the total cost to the purchaser is the $1,010 plus the $7.50, or $1,017.50.

An investor purchased 100 shares of ABC common stock valued at $6,000. What is the adjusted cost basis per share of this position after the company pays a 20% stock dividend?

The total value of the initial position is unchanged, remaining at $6,000. After the stock dividend, the investor owns 120 shares (100 × 20% = 20 + 100 = 120). Therefore, the adjusted cost basis is $50 per share ($6,000 divided by 120 = $50). It is important to remember that anytime there is a distribution resulting in additional shares (stock split, stock dividend), the cost basis per share is reduced while the total account value remains the same.

An investment company registered under the Investment Company Act of 1940 that allows shareholders to sell their shares back to the company at the net asset value per share only at certain specified times is

The unique characteristic of an interval fund is that at certain specified intervals, shareholders are able to sell their shares back to the fund at NAV. True, these are closed-end investment companies, but on the exam, you always need to choose the most specific answer. Open-end investment companies (mutual funds) and unit investment trusts (UITs) permit redemption at NAV on a continuous basis, not at specified intervals.

On Monday, June 1, an investor pays 92 to purchase a 5% J&J municipal bond maturing on July 1, 2030. Purchasers of bonds pay accrued interest to the seller, in addition to the market price of the bond. How many days of accrued interest will this seller receive?

There are 152 days of accrued interest. On municipal bonds, the accrued interest calculation uses 30-day months and 360-day years. Interest begins to accrue on the last interest payment date and runs up to, but not including, the settlement date. This J&J bond pays interest on January 1 and July 1 of each year. Therefore, with a June purchase date, the most recent interest payment was on the previous January 1, the day that interest begins to accrue. The trade date is June 1 with a settlement date of June 3 (T+2). The buyer of the bond becomes the owner of the bond on June 3, and from that date forward, the buyer is entitled to the interest. That is why interest payable to the seller stops accruing on June 2. Here is the math. We have 5 months (January, February, March, April, and May) plus 2 days of accrued interest in June. With each month counting as 30 days, that is 150 days + 2 more in June equaling 152 days. How do we know that the J&J dates are the first of the month rather than the 15th? Good question. For the answer, we look to the maturity date of the bond. That is July 1, 2030, and is the clue that the interest payment dates are on the first of the month.

Which of the following is not a characteristic of hedge funds?

They offer managers high fixed fees. Hedge funds attempt to attract the top managers because they offer performance-based fees, which vary based on fund performance. The typical fee structure is 2% + 20%, where 2% is the fixed fee and 20% of the profits is the performance portion.

One of your existing clients wishes to open a new account in the name of his spouse and enter orders on her behalf. Which of the following statements is true?

This action is prohibited unless the spouse signs a trading authorization.

A customer in the 28% tax bracket wants to buy a municipal GO bond with a 7.5% yield that matures in 6 years. The tax-equivalent yield of this bond is

To calculate the taxable return, use the tax-free equivalent yield formula: municipal bond yield ÷ (1 − investor's tax bracket). Using this formula, 0.075 ÷ (1 - 0.28) = 0.104, or 10.4%. This means the investor, who is in the 28% tax bracket, must earn 10.4% in taxable interest to equal the 7.5% tax-free municipal interest yield.

A 7% convertible debenture is selling at 101. It is convertible into the common stock of the same corporation at $25. The common stock is currently trading at $23. If the stock were trading at parity with the debenture, the price of the stock would be

To determine the parity price of the common, first find the number of shares the debenture is convertible into (conversion ratio) by dividing par value by the conversion price ($1,000 / $25 = 40 shares). Next, divide the current price of the bond by the conversion ratio. The result is the parity price of the common stock. (1,010 / 40 = $25.25).

XYZ Widgets is a publicly traded corporation. Upon the death of one of the founders of the company, a donation of 100,000 shares of XYZ stock is made by the executor of the deceased's estate. This would now be considered

Treasury stock is issued and outstanding stock of the corporation that has been reacquired by the company. In some cases, such as this question, the stock is donated back to the company, but in most cases, it has been purchased by the company in the open market. Regardless of how stock is acquired, it is no longer outstanding, and it is held in the company's treasury

An aggressive investor buys ABC stock with a beta of 1.7. The S&P 500 has a 10% rate of return for the year, and ABC's return is 12%. What is the alpha for ABC?

With a beta of 1.7, an investor would expect ABC stock to be 70% more volatile than the general market as measured by the S&P 500. Remember, the beta of the market is 1.0. Therefore, we would expect to see the stock return 17% based on the S&P 500's 10% return. However, the actual return on ABC was only 12%. Alpha measures the difference in the actual return vs. the expected return. The difference between the expected return (sometimes referred to as the required return) of 17% and the actual return of 12% is a negative 5%. That represents an alpha of -5% for ABC stock.

One of the features of variable insurance products is the ability to withdraw money from the policies. Which of the following statements is correct?

Withdrawals from variable annuities are taxed on a LIFO basis, while those from variable life are taxed on a FIFO basis. One advantage to withdrawing cash value from a variable life insurance policy is that it receives FIFO treatment. That means there is no tax until the withdrawal reaches the cost basis (premiums paid) of the policy. With annuities, the taxation is LIFO. Therefore, the first money withdrawn is taxable. In addition, if the policyowner is not yet 59, the 10% penalty applies. **This question deals with material not covered in your LEM, but it relates to recent rule changes and/or student feedback.

Which of the following transactions would be acceptable investments for a pension fund?

Writing a covered call Writing a covered call has less risk than writing a naked option. A covered call writer is merely using options to increase the income on his portfolio. Fiduciaries, such as those who invest for pension fund portfolios, should avoid risky transactions.

Which of the following would be considered a bullish strategy?

Writing a put Those who write put options benefit when the price of the underlying asset increases (bullish). It is just the opposite for those who write a call. Spreads are bearish when the low strike price is sold and the high strike price is bought. That results in a debit when it is a put spread and a credit when it is a call spread. Credit put spreads and debit call spreads are bullish because it is the low strike that is purchased and the high strike that is sold.

An investor would like to make an intermediate-term investment in a debt security whose duration is equal to its maturity. Which of the following AAA-rated bonds should the registered representative recommend?

XYZ zero-coupon bond maturing in five years Because they make no interim payments, zero-coupon bonds have a duration equal to maturity. Of the choices offered, only XYZ is both intermediate-term (2 to 10 years to maturity) and zero-coupon.

A bond is currently priced at 96. Which of the following yields would be the highest?

YTM. When a bond is selling at a discount (96 means 96% of par value, or $960), the investor's yield is greater than the stated coupon or nominal yield (those two terms are synonymous). Because the yield to maturity considers the $40 profit to be made when the bond matures, the YTM is higher than the current yield.

A registered representative places an ad in her church newsletter promoting her services as an expert retirement planner. Does this advertisement need to be preapproved by a principal?

Yes, the ad will be defined as retail communication, which requires preapproval of a principal. Although it isn't specified that the ad will be received by more than 25 retail investors during a 30-day period, the best answer is the ad will be defined as retail communication requiring preapproval of a principal. How retail communication is delivered (mailed as opposed to handed out) in this question is not relevant; whether promoting a service or product, either would need preapproval, and the fact that the ad is placed in a church newsletter provides no exemption from preapproval requirements.

An investor purchased 10 GO bonds at a discount of 2 points per bond. The bonds mature in 10 years. After holding the bonds for 5 years, they were sold at par. For tax purposes, the investor has

a $100 gain. The cost per bond is $980. The accretion amount each year is $20. $20 ÷ 10 years = $2 per year. $2 per year × 5 years = $10 per bond accretion, making the adjusted cost basis $990 per bond. When the bonds are sold at par ($1,000), there is a profit of $10 per bond × 10 bonds, which equals a $100 gain.

At year's end, your client reports $12,000 in capital gains and $20,000 in capital losses. The net effect of this on his taxes would be

a $3,000 deduction from ordinary income with a $5,000 loss carryforward. The customer may offset all of the gains with the losses. This leaves a new loss of $8,000. Because the maximum net capital loss that may be deducted against ordinary income is $3,000 per year, we take off the $3,000 and have a carryforward of the balance ($5,000).

An example of a taxable bond issued by a municipal government is

a Build America Bond (BAB). BABs are municipal issues created under the Economic Recovery and Reinvestment Act of 2009 to assist in reducing the cost of issuing municipalities and to stimulate the economy. Bonds to fund municipal projects have traditionally been sold in the tax-exempt arena, but BABs are taxable obligations.

A municipal bond is purchased in the secondary market at 102½. The bond has five years to maturity. Two years later, the bond is sold for 102. The tax consequence to the investor is

a capital gain of $5 per bond. Municipal bonds bought at a premium, either in the new issue or secondary market, must be amortized. The amount of the premium is 2½ points, or $25. As the bond has five years to maturity, the annual amortization amount is $5 per bond. After two years, the bond's basis has been amortized down to 101½. At that point, a sale at 102 generates a capital gain of $5 per bond.

If an investor wishes to open a cash account in her name only and allow her spouse to make trading decisions as well as withdraw cash and securities, she must instruct her broker-dealer to open

a cash account with full power of attorney.

Most business development companies (BDCs) are classified as

a closed-end investment company. A major difference between BDCs and the other investment companies is the active role played in the management of the businesses in the portfolio. That is what business development is about helping smaller businesses develop into larger ones.

You have been given the name of a new potential client who responded to a marketing piece sent out by your broker-dealer. Which of the following would be the most appropriate way to obtain information about the client's objectives and investment constraints?

a face-to-face meeting at client's house

Benefits of a municipal bond advance refunding include

a higher rating and greater marketability. Advance or prerefunding is refinancing an existing municipal bond issue before its maturity or call date by using money from the sale of a new bond issue. Because the proceeds of the new issue are placed into special U.S. government securities, the rating is automatically at the top. The higher rating increases the marketability. The current bond still exists until the specified call date. As such, the coupon has not changed. There are no taxes to be saved.

A characteristic of hedge funds that would not be found in a mutual fund is

a lock-up period. Hedge funds generally employ a lock-up provision. This is to ensure that capital invested by shareholders will remain with the fund long enough for the manager to implement the intended fund strategy. There is no standard lock-up period; it can differ from fund to fund. It should always be noted that during the lock-up period, the investment is essentially rendered illiquid. Hedge funds and mutual funds have professional management and diversified portfolios. Although hedge funds can use margin in portfolio transactions, they, like mutual funds, cannot be purchased on margin.

All of the following statements about variable annuities are true except

a minimum rate of return is guaranteed. The return on a variable annuity is not guaranteed; it is determined by the underlying portfolio's value. Variable annuities are designed to combat inflation risk. The number of annuity units becomes fixed when the contract is annuitized; it is the value of each unit that fluctuates.

When a broker-dealer makes a market, it is acting as

a principal. Making markets is a principal activity. The broker-dealer stands ready, willing, and able to buy or sell securities for its own account. A dealer acts as a principal when it owns the securities it trades. When the broker-dealer is not acting for its own account, it acts as a broker or an agent.

Under FINRA's rules governing the activities of member broker-dealers, prior notification to the employing firm and prior written consent from the employing firm would be required to open a brokerage account for all of the following except

a registered representative of another member opening a 529 plan. FINRA requires prior written notification be made and prior written consent be received before any employee can open a brokerage account with other members or financial institutions. Exceptions include accounts where the only activity will be in 529 plans, mutual funds, or variable annuities.

Instead of signing on the back of a security sold, the registered owner could sign on a separate paper called

a stock (or bond) power. If the client were to sign the back of the certificate, that security would now be completely negotiable. If lost, it would be the same as losing an endorsed check. To minimize problems, make the assignment on a stock power, which is a separate piece of paper, and when it is put together with the actual certificate, it is treated as if the certificate itself had been signed.

Each of the following is a defined contribution plan except

a stock option plan. Money-purchase pension plans, 401(k) plans, and qualified profit-sharing plans are all examples of defined contribution plans. An employer may offer stock options that give an employee the right to purchase a specified number of shares of the employer's common stock at a stated exercise price over a stated time period. No actual contribution is made, just payment when the employee decides to exercise the option. Unlike the other choices, this is not a qualified plan.

In compliance with the Securities Act of 1933, a prospectus is always required when selling interests in

a unit investment trust. A unit investment trust has a one-time offering of its securities. After that, there is no public secondary market trading (the sponsor may engage in secondary trading solely to redeem units). Therefore, as a public offering of a new security, a prospectus is required when attempting to make a sale. Closed-end funds only require a prospectus on their initial offering. Once that is completed, all transactions are in the secondary market and no prospectus is used. Both municipal bonds and Treasury securities are exempt from the prospectus requirements of the Securities Act (although municipal bonds generally need an official statement).

Options communications may contain projected performance figures (including projected annualized rates of return), provided that

all such communications are accompanied or preceded by the Options Disclosure Document. Once the communications get specific enough to include performance figures, prior or concurrent delivery of the ODD is necessary. Although the risks must be disclosed, possible defalcation by an exchange officer (a form of embezzlement), is not an investment-related risk. The requirement to return the options account agreement within 15 days has nothing to do with permitted options communications. When annualized rates of return are shown, the minimum is a 60-day period, not 30 days.

Fee-based accounts would tend to be most suitable for investors who follow

an active approach to investing. Because one of the key benefits of a fee-based account is the elimination of transaction fees (commissions), it is most suitable for those who are active traders, such as those who take a tactical approach to investing. The other choices engage in far less trading activity, potentially not being able to take full advantage of all of the benefits of the fee-based program.

An exchange-traded fund whose strategy is to generate performance opposite that of the designated index is called

an inverse fund. Inverse ETFs (also called short funds) seek to deliver the opposite of the performance of the index or benchmark they track. There are some who call these reverse funds, but the SEC, FINRA, and NASAA do not use that term. Leveraged ETFs seek to deliver multiples of the performance of the index or benchmark they track. There are leveraged inverse funds, but the term inverse would have to be in the description. Hedge funds are not exchange traded.

All of the following are liquidity measures except

annual dividends per share divided by earnings per share. Liquidity is the ability to convert an asset into cash. All of the options listed are used to measure the liquidity of a company except for the dividend payout ratio which would tell you nothing about liquidity.

If an OTC market maker provides a firm quote to another broker-dealer then refuses to buy or sell at the price quoted, the market maker is said to be

backing away. A dealer that does not honor its quote is said to be "backing away." Interpositioning is the unethical act of a member firm placing a third party between itself and the best available market. Freeriding is a term used when securities are purchased and then sold without making payment for the purchase on settlement date. Crossing quotes is a term that would only appear on the exam as an incorrect choice (as it does here).

A customer opens an account, and payment and delivery instructions are established. Beyond the opening of the account, these instructions may

be changed for individual transactions, or going forward, for all transactions. Once payment and delivery instructions are established at the time the account is opened, they can be changed for any individual transaction or for all transactions going forward.

A customer is long an ABC Apr 40 call and is short an ABC Jul 40 call. Which of the following best describe his position?

bearish and calendar spread The July call will have a higher premium than the April call because it has more time value. Because the customer is selling the call with the higher premium, he is counting on the July call to go unexercised, which would allow him to keep the premium as a profit. That means the market value of the underlying security must either stay the same or decline. Therefore, this customer's position is bearish. Because the options expire in different months, the trade is a calendar spread.

A corporation has determined that if it were to go bankrupt, common stockholders would receive $8.47 per share. This calculation is known as

book value per share. The liquidating value of the company is its book value; the book value per share is what the common stockholders would receive.

If your customer owns 100 shares of a volatile stock and wants to limit downside risk, you may recommend

buying puts. Downside risk is reduced by purchasing a put with a strike price at or close to the stock's purchase price. Should the stock decline below the strike price, the investor can exercise the put at the strike price. Selling put options will increase the downside risk. Buying calls is a bullish strategy that increases downside risk. Shorting stock will lock in the current price but will limit upside potential.

Features of an employee stock purchase program (ESPP) include all of the following except

contributions are made with pre-tax dollars. Employee stock purchase plans (ESPPs) are not qualified plans. That means that the employee purchases the stock with after-tax dollars. For example, an individual has a monthly salary of $5,000 and elects to contribute 10% of gross salary to the plan. The employer will take $500 per month out of the paycheck after subtracting withholding tax and Social Security contributions and any other deductions. Before enrolling in the plan, this employee's monthly take-home pay might have been $3,700. Now it will be $3,200.

All of the following ratios are measures of the liquidity of a corporation except

debt/equity ratio. Liquidity ratios measure a firm's ability to meet its current financial obligations and include the current ratio and acid test (quick) ratio. However, the debt/equity ratio is a capitalization ratio and measures the amount of leverage compared to equity in a company's overall capital structure.

On exercise of the option, the holder of a long call will realize a profit if the price of the underlying stock

exceeds the exercise price plus the premium paid. To profit on a long call, the market price must exceed the strike price plus the premium paid (the breakeven point) computed using the call up rule.

A client of a member broker-dealer is disgruntled with the attitude of some of the firm's staff. A complaint is sent by email to the manager of the branch servicing the account. FINRA rules require that a record of this complaint be maintained for no less than

four years. FINRA rules require that written complaints (and a complaint filed electronically is considered written) be kept for a minimum of four years. This is the only four-year FINRA requirement you are apt to see on the exam. The MSRB rule on retention of customer complaints is six years so be sure to keep these two straight because both are testable.

A corporate bond is quoted in the Wall Street Journal as follows: Bid: 100½ Asked: 100¾ Bid Chg.: -⅛ Yield: 5.75 From this information, you know the nominal yield is

greater than 5.75%. The bid and asked prices show that the bond is being quoted at a premium (above par), with a yield to maturity of 5.75%. When bonds are trading at a premium, the nominal yield (coupon rate) is greater than the yield to maturity.

A Treasury bond is quoted in The Wall Street Journal as follows: Bid 100:15 Asked 100:17 Bid Chg. -1 Yield 7.9 From this information, you know that the nominal yield is

greater than 7.90%. The Bid and Asked prices show that the Treasury bond is being quoted at a premium (above par), with a yield to maturity of 7.9%. When bonds are trading at a premium, the nominal yield (coupon rate) is greater than the yield to maturity.

An 18-year-old, unmarried high school student sought a safe investment for a $30,000 bequest until after she graduated from college. Her intent was to use the funds for the down payment on a house after graduation. Her agent recommended she choose a variable annuity as a safe haven for the funds. This recommendation is unsuitable because

her situation exposes her to surrender charges and early withdrawal penalties The funds are not liquid due to the surrender fees, and there is also a 10% penalty on withdrawals of earnings before age 59½.

For which of the following would the net revenue-to-debt service ratio be applicable?

hospital bonds This is the coverage ratio. Because revenue bonds are only backed by funds generated by a specific source, it is important that net revenues exceed debt service requirements. Hospitals are often built with the proceeds of revenue bond issues.

When part of an issue of speculative bonds with a 25-year maturity are called, the effect on the remaining bonds will be to

improve their quality. Speculative bonds are those with lower ratings. They are considered to be of lower quality because the risk of timely payment and principal are higher than investment-grade bonds. When a company shows its determination to honor its debt by paying off some of it in advance, the rating associations take note of that and invariably increase the rating. Compare this to your personal credit score. Your score might be relatively low because you have a lot of outstanding debt. As you pay down that debt, your credit score is likely to increase. It is the same logic here.

Which of the following describes the position in a call option on a Swiss franc with a strike price of 120, a premium of 7, and a current market of 126?

in the money In this case, the strike price is less than the market price, so a call option would be in the money by the difference between the strike price and the market price (six points, in this case). At the money means the strike price and the market price are the same. At parity means the premium equals the intrinsic value.

Under Options Clearing Corporation (OCC) rules regarding options communications with the public, if an educational piece making no projected performance figures or recommendations is distributed to customers, it

need not be preceded by an options disclosure document (ODD).

Among the differences between a real estate investment trust (REIT) and a real estate limited partnership investment (a DPP) is that

only the DPP is a flow-through vehicle.

All of the following option strategies could be effectively used in a bear market except

short straddle Short straddles are appropriate only in flat or neutral markets. The writer will lose in a rising market (the call will be exercised) or a falling market (the put will be exercised). Short calls and short call spreads are bearish, as are debit (long) put spreads.

The LLAW Manufacturing Company issued a 6.25% debenture 5 years ago. The bond is callable in seven years at 102 and matures in 15 years. The bond's current yield is 4.23%. If one of your customers decided to purchase this bond, they would have to understand they would be

paying a premium for the bond. The first thing to notice is that the current yield is below the nominal (coupon) rate. That automatically tell us the bond is selling at a premium. Whenever a bond is selling at a premium, in increasing order, the order of the yields is yield to call, yield to maturity, current yield, and nominal yield. Therefore, if the current yield is 4.23%, the yield to mature cannot be higher than that; it must be less. As we say in the LEM, "if you pay more, you get less" and "if you pay less, you get more." The issuer pays the call price when, and if, the bond is called.

For several months, there has been no activity in a client's account. A monthly statement would be required if the account contained

penny stocks. SEC rules require that monthly statements be sent to a customer when there are penny stocks in the account. Otherwise, statements for all accounts are sent quarterly. In the case of DPPs and unlisted REITs, an estimated value of the security must be provided.

Greater Growth Capital (GGC), a FINRA member firm, has just been acquired by Better Retirement Outcomes (BRO), a much larger FINRA member. If GGC would like to effect a bulk transfer of its customer accounts using a negative consent procedure, FINRA rules

prohibit GGC from charging a fee to any existing GGC customers who decide to transfer their accounts to a different firm.

A registered representative would recommend a customer establish a short straddle on T-bonds when interest rates are expected to

remain unchanged Any straddle writer is always looking for a stable market. Volatility is the biggest enemy of the writer. Because this question is referring to debt options, their price movements are based upon changes in interest rates. No fluctuations in interest rates means no price changes.

If interest rates increase, the interest payable on outstanding corporate bonds will

remain unchanged The interest payable is the nominal yield, which is stated on the face of the bond. It is the percentage of face value the bond will pay each year regardless of the prevailing interest rates in the market. It is the market price of bonds, not the interest payable, that responds inversely to changes in interest rates.

Which of the following positions subject an investor to unlimited risk?

short naked call and short sale of stock Short stock and short naked calls subject an investor to unlimited risk because there is no limit on how high a stock's price might rise. Risk is limited for the other positions.

A prospect is heavily invested in the common stock of an employer's company, ABC, relative to other investments. The stock has performed well over the last 15 years and the prospect is very happy with the investment. After reviewing financial and nonfinancial criteria, you have determined that

selling a portion of ABC and using the proceeds to purchase mutual funds will reduce his nonsystematic risk. This prospect is exposed to a significant amount of business (nonsystematic) risk, as indicated by the large investment in ABC common stock. Business risk can be reduced by diversifying the portfolio; therefore, recommending the sale of a portion of the ABC stock and using the proceeds to purchase mutual funds is suitable. There will be tax considerations, but the use of FIFO accounting will likely expose the prospect to higher capital gains taxes than other accounting methods and may not be the best approach to liquidation.

Once the IRS determines that a tax shelter is abusive, it may do all of the following except

sentence the abuser to a prison sentence.

All of the following option contracts are in-the-money when XYZ is 54 except

short XYZ 50 put. Call options are in-the-money whenever the market price is greater than the strike price. Put options are in-the-money whenever the market price is lower than the strike price. Try to remember: call up and put down.

For a regular standardized option, any gain on the sale of the contract is

short-term cap gains Regular standardized options have a maximum expiration of 9 months, so a gain on these types of contracts can only be short term for tax purposes. It is only the LEAPS options where it is possible to hold a long option position for more than 12 months. That is the only case where a long option can realize long-term treatment. The question is dealing solely with the options contract, not the underlying security.

An investor buys two ABC Nov 50 calls, three ABC Dec 45 calls, and one ABC Jan 50 call. The best way to describe the portfolio is that it consists of

six options of the same class. A class of options is when they are all of the same type (in this case, calls) and all on the same underlying security (in this case, ABC). Yes, they are all of the same type, but, in a question like this, FINRA is asking for the most specific answer. Same class is more specific than same type. The same series would be if they all had the same expiration date and exercise price.

Your client has purchased shares of VACL at several different times. A view of the client's account ledger indicates the following: 100 shares @$50 on February 12 100 shares @$52 on April 23 200 shares @$49 on May 12 100 shares @$55 on June 28 The client decides to sell 200 shares of the VACL on November 14 of the same year when the price of the stock is $53 per share. Tax consequences would be minimized if the investor

sold the shares purchased in June at $55 and the shares purchased in April at $52. By using the identified cost method, the investor would sell the highest cost purchases. This would result in the lowest taxable gain (or perhaps even a loss). Average cost is only available for mutual funds.

For U.S. investors holding American depositary receipts (ADRs), dividends received are

subject to a foreign withholding tax. Any tax taken on dividends received from ADRs is taken in the country of origin. This is a foreign withholding tax for U.S. investors. The foreign withholding tax may later be taken as a credit against any U.S. income taxes owed by the U.S. investor.

Due to a sudden drop in earnings, the board of directors of Amalgamated Metal Industries (AMI) has voted to suspend all dividend payments this year. This would have the least effect on holders of AMI's

subordinated debentures. Regardless of the level of seniority of a preferred stock, it comes behind any debt security. More importantly, interest on a debenture, subordinated or not, is a contractual obligation. Unlike the dividends on stock, the decision to pay or not to pay interest is not an optional one. Failure to pay interest on a debt security can lead to foreclosure and bankruptcy proceedings.

The call provisions of a municipal issue would be detailed most completely in

the bond resolution. The bond resolution is the document that authorizes the issuance of a municipal bond. The resolution also describes the proposed issue's features and the issuer's responsibilities to its bondholders.

All of the following statements regarding the short sale of a listed security are true except

the buyer must be advised that he is purchasing borrowed shares. On an exchange floor, short sales can be effected at any time in the trade sequence. In addition, short sales may be effected at either the opening or closing. The buyer is never informed that shares being purchased represent borrowed shares.

FINRA Rule 2330 states: no member or person associated with a member shall recommend to any customer the exchange of a deferred variable annuity unless such member or person associated with a member has a reasonable basis to believe the exchange suitable, taking into consideration whether

the customer has had another deferred variable annuity exchange within the preceding 36 months. FINRA Rule 2330 specifies the suitability conditions surrounding the recommended exchange of an existing deferred variable annuity contract for a new one. Included in the list of considerations is determining if the customer has made another variable annuity exchange within the previous 36 months. Unlike life insurance, where the insured's health is important, there are no health questions on an annuity application. It is safe to assume that any annuity exchange will be done under the provisions of Section 1035. This means there are no tax consequences. The assumed interest rate is for internal purposes; it is not a differentiator when deciding which variable contract to purchase.

If a customer of your firm receives stock from the estate of her mother, the stock's cost basis in the hands of the customer is

the market value at date of death. When securities are inherited, the heir receives a cost basis calculated as of the deceased party's date of death.

A corporation's income statement reports net income of $10 million for the year. The company has one million shares of 4% $50 par value preferred stock and two million shares of common stock. If the corporation paid a quarterly dividend of $0.60 per share of common stock,

the dividend payout ratio was 60%. The dividend payout ratio is the percentage of the net income (after preferred stock dividends) paid out to the common shareholders. The net income is $10 million. The preferred dividend is $2 per share ($50 par times 4% = $2). With one million shares, the total preferred dividend is $2 million (1 million shares at $2 per share). Because the preferred dividend must be paid before any earnings are available to common stockholders, we subtract that $2 million from the net income. That leaves $8 million in earnings available to common. There are 2 million shares receiving an annual dividend of $2.40 ($0.60 quarterly). That means $4.8 million of the $8 million available is paid, or a ratio of 60% ($4.8 million ÷ $8 million = 60%). Or, the earnings per share is $4.00 ($8 million divided by 2 million shares) and $2.40 in dividends paid out of $4.00 earnings made is 60%. The preferred stock is paying a dividend of 4% of the par value, but that does not tell us the current yield. To know the current yield, we must know the current market price of the stock and the question does not supply that value.

The bond resolution includes all covenants between

the issuer and the trustee acting for the bondholders.

The unqualified legal opinion on a municipal bond states that

the issuer has the authority to issue bonds that are legal, valid, and enforceable obligations of the issuer. An unqualified opinion means the bond counsel attests that, to the best of its knowledge, the issuer has the legal right to issue the securities in question. In the case of tax-exempt bonds, the interest the issuer will pay on the bonds is exempt from federal taxation and the bonds are exempt from federal registration requirements. The legal opinion does not evaluate the issue's marketability, or safety, debt service requirements.

Marking-to-the-market is

the revaluing of securities held long or short in the account based on the actual CMV of the securities. In order to determine the necessity of additional maintenance calls or the availability of SMA, this revaluing is done at least once per business day in accounts that have margin balances.

If an investor purchased a municipal bond in the secondary market, which of the following would not be a factor in calculating the total dollar amount paid for the bond?

the scale The scale, or reoffering scale, represents the prices and/or yields (listed by maturity date) at which new issue securities are offered for sale to the public by the underwriting syndicate. Because this question refers to a secondary market purchase, the scale would not apply. When computing the total price of a bond purchase, we need to know the accrued interest. Because interest is computed up to—but not including—the settlement date, clearly that is required information. The dated date is the date from which interest begins to accrue. True, this is only applicable for the first interest payment, but the question doesn't specify that this bond has already made that payment. Remember, secondary market transactions can take place the same day the new issue is released. Finally, it would be impossible to compute the accrued interest without knowing the interest rate being paid by the issuer. There is an important test-taking tip here. Scale has not appeared in the material yet, so we do not expect you to know what it means. But you should know that the other three items are necessary to compute the total purchase price. By process of elimination, scale must be the correct choice. This logic can be helpful on the real exam.

An analyst reports that a stock's price is consolidating. This means

the stock's trendline is moving primarily in a horizontal direction. In general, when the trendline of a stock's market price is moving within a very narrow range (the chart is basically a pattern of horizontal movement), the technician views that as a consolidation. Within a relatively short time after the consolidation has been verified, it is expected the price will move. What isn't determined yet is if the movement will be up (bullish) or down (bearish).

If a customer writes one uncovered in-the-money put, the maximum loss to the customer is

the strike price minus the premium multiplied by 100 shares. If the stock becomes worthless, the investor will be forced to buy the stock at the strike price, but they still keep the premium received when the option was written. Essentially, maximum loss is breakeven multiplied by 100 shares. The fact that the put was in-the-money when the option was written is of no consequence. It is only there to distract you.

All of the following are reasons for entering a stop order except

to guarantee execution at a specified price or better. A stop (loss) order is entered to protect a profit or to limit a loss. Execution at a specific price can never be guaranteed because a stop order becomes a market order when the stop price is hit. It is only a limit order (if executed) where the specified price (or better) is guaranteed.

A member firm's client has issued instructions for the assets held in an account at another member to be transferred to this account. The member firm has received the proper ACATS validation from the carrying firm. Therefore, it is expected that the transfer will be completed within

two additional business days. When using the ACATS system, validation takes place in one business day and the transfer in three business days. Please note the question states that validation has been received. That means the transfer will take place in two more business days (making it a total of three business days). Follow this trail. On Monday, the receiving firm sends the TIF to the carrying firm. On Tuesday, the carrying (delivering) firm validates the TIF and has three business days to make delivery (by Friday). On Wednesday, the validation gets to the receiving firm. Yes, the exam can be that tricky.

A registered representative is contacted by a new client who wishes to transfer his 401(k) plan assets into an IRA at your firm. The amount being transferred is $1,050,000 and the registered representative recommends diversifying into Class C shares of five different funds in the XYZ mutual fund group. This recommendation would be

unsuitable because a purchase of that size is likely going to reach a significant Class A share breakpoint. There are two problems with recommending Class C shares for this client. The first is that the no-front-end-load feature is not a selling point. A typical load on a $1 million purchase of Class A shares is zero, eliminating any benefit of the Class C share. The second point is that Class C shares generally have a higher 12b-1 charge and, even if the fund group was one of a small number that carries a 1% load on this large of a purchase, it wouldn't take long for the lower expense ratio of the Class A shares to result in a better deal for the investor. As a test-taking tip, beware of large purchases of Class C and especially Class B shares ($100,000 or more).

A municipality that has issued grant anticipation notes (GANs) (short-term municipal notes) does so in expectation that the debt service will be paid by the receipt of funds attained

via grants from the federal government. GANs are short-term municipal notes issued in anticipation of funds via grants that the municipality is expecting from the federal government.

Which of the following equity securities has the longest expiration date?

warrants - Warrants commonly have expiration dates that are one, five, or even more years in the future. Preemptive rights generally expire in a maximum of 45 days, and common and preferred stock do not have expiration dates.


Ensembles d'études connexes

Government Unit 3 Questions and answers

View Set

Senderos 1-1 Estructura 1.1 NOUNS

View Set

PRAXIS 5001 - MULTIPLE CHOICE SAMPLE QUESTIONS (ENGLISH)

View Set

21&22 A&P II Martini Study Guide

View Set